Sunteți pe pagina 1din 98

Self-Administered QEI-MCQ Exam 1

Section: 1

Question: 1

A 50-year-old man requests information concerning prostate cancer. Which one of the following statements is
true regarding prostate cancer?

There is an increase in mortality rate that closely follows the increase in incidence rate.
The lifetime risk of prostate cancer is one in five.
His annual risk of being diagnosed with prostate cancer decreases with age.
Prostate cancer now leads lung cancer as a cause of mortality in Canadian men.
Prostate cancer is the most frequently diagnosed non-dermatological cancer in Canadian men.

Question: 2

A 30-year-old woman is referred from the Emergency Department for an intravenous pyelogram (urogram) to
investigate abdominal pain and hematuria. She is in considerable pain and anxious to find the cause of her
symptoms. The radiologist should proceed

only if consent for "investigations and treatment" has been signed on the emergency admission
record.
if her husband or a first-degree relative has signed a consent form for an intravenous pyelogram.
because of implied consent by her willingness to undergo further investigation.
without the need for consent because this is an emergency procedure.
only after the risk of serious complications from contrast media has been explained to her.

Question: 3

A randomized placebo-controlled trial of a new agent for Acute Respiratory Distress Syndrome in the
Intensive Care Unit is proposed. Patients will be randomized tocurrent therapy plus placeboorcurrent therapy
plus the new agent. In this setting,which one of the following is most likely to help decide whether this trial
should proceed or not?

The trial will enroll only adult patients.


The trial will only enroll capable patients.
The trial will only enroll less critically ill patients.
The trial addresses a condition for which current therapy is often ineffective.
The trial drug must be known to be without risk.
Question: 4

A 42-year-old woman presents to your office complaining of feeling sad and distant from her family over the
past 2 months. She admits to crying spells when alone and difficulty falling asleep. She continues to work part
time and still enjoys gardening. She started taking St. John's Wort 300 mg 3 times a day one week ago as
recommended by her naturopath Although she had "the baby blues" following the birth of her last child 13
years ago there is no previous or family history of depression or other psychiatric disorder. She denies suicidal
ideation. She would prefer staying on her current medication. In addition to follow up, which one of the
following is the most appropriate management at this time?

Stop St. John's Wort and begin venlafaxine (Effexor XR) 37.5 mg daily.
Stop St. John's Wort.
Maintain St. John's Wort at current dose.
Increase St. John's Wort to 1800 mg day.
Continue St. John's Wort at current dose and add zopiclone (Imovane) 7.5 mg at bedtime

Question: 5

A 52-year-old woman is admitted to hospital for hysterectomy for adenomyosis. Her body mass index is 30.
You order perioperative heparin and counsel your patient that:

The length of the surgical procedure will not affect her risk
The heparin should be continued for 6 weeks postop
The highest risk of thromboembolic complications occur within the first 72 hours postoperatively
The risk is not affected by the finding of a uterine sarcoma
Pulmonary emboli usually originate from the pelvic veins

Question: 6

A 13-year-old child presents to the Emergency Department 1 hour after a soccer injury to her knee.
Examination reveals moderate, non-tense swelling, and is otherwise limited due to patient anxiety and
generalized pain. Radiograph examination is unremarkable. Which one of the following is the most
appropriate management of this child at this time?

Magnetic resonance imaging examination.


Aspiration and pressure dressing.
Long leg cast.
Diagnostic arthroscopy.
Physiotherapy.
Question: 7

A 70-year-old man is brought to the Emergency Department with a history of confusion, irritability and partial
loss of consciousness for the past 3 hours. He apparently fell down the basement stairs 1 week ago, struck his
head and was unconscious for about 10 minutes. He is now stuporous and his level of consciousness is
decreasing. Which one of the following is considered to be the most likely cause of his deterioration?

Subdural hematoma.
Subarachnoid hemorrhage.
Cerebral thrombosis.
Cerebral contusion.
Transient ischemic attack.

Question: 8

A 17-year-old woman presents to the Emergency Department because of severe vulvar burning and inability to
void. Examination reveals a mixture of small, fluid filled vesicles and yellow ulcerated lesions on the vulva
and around the urethra. Which of the following tests is the most appropriate to confirm your diagnosis?

Biopsy of the lesions


Urine culture
Viral culture of the lesions
Venereal Disease Research Laboratory (VDRL)
Viral serology

Question: 9

You are asked to see a 50-year-old man 2 hours after a laparotomy for gastric resection. His P aCO2 is 50 mm
Hg and his PaO 2 is 75 mm Hg on 4 liters/min with nasal prongs. The most likely reason would be which one
of the following?

Normal recovery from an inhalation anesthetic


Metabolic alkalosis
Respiratory insufficiency
Postoperative hypermetabolic period
Compensatory respiratory alkalosis
Question: 10

A 5-year-old girl has a history of recurrent vomiting since birth. She now complains of small amounts of dark
blood in the vomitus and of difficulty eating solids. Her hemoglobin is 101 g/L. Which one of the following
is the most likely diagnosis?

Reflux esophagitis.
Duodenal ulcer.
Diverticulum of the esophagus.
Bezoar.
Hiatus hernia.

Question: 11

A 9-year-old boy is seen in the outpatient clinic for lethargy, poor academic performance, constipation, and
slow growth of 12 months' duration. Which one of the following diagnoses is the most likely?

School phobia.
Diencephalic syndrome.
Hypothyroidism.
Panhypopituitarism.
Diabetes mellitus.

Question: 12

A 32-year-old woman presents with fever, headache and a stiff neck. Cerebrospinal fluid shows:

70 X 106/L
White blood cell count
92% polymorphonuclears
Protein 1.5 g/L
Glucose 1.5 mmol/L

These findings are most compatible with meningitis due to

a virus.
tuberculosis.
meningococcus.
Cryptococcus.
Listeria.
Question: 13

The best plan of management of a pregnant woman with a past history of herpes simplex genital infection is

vaginal delivery.
cervical culture at 2-week intervals from week 32.
observe for lesions at the time of labor.
prophylactic topical acyclovir.
elective cesarean section.

Question: 14

In a 1993 decision regarding a request for physician-assisted suicide, the Supreme Court of Canada

decided there were compelling social reasons to infringe on the patient's constitutional rights.
relied on the law which makes suicide a crime in Canada.
made an exception based on the specific circumstances of the patient's situation.
decriminalized physician-assisted suicide.
rejected arguments that suffering would result from their ruling.

Question: 15

A 17-year-old girl suffering from depression is treated with sertraline. She does not show any improvement,
but after 2 months she tells you that she has been taking amphetamines for a year. Which one of the following
is the most appropriate management?

Discontinue sertraline and arrange to see the patient in follow-up.


Discontinue sertraline and start paroxetine.
Refer her to a colleague because she has misled you.
Give her written information on drug abuse.
Inform her parents that you need their assistance.
Question: 16

Several months afterattempting to resuscitate a patient in the Emergency Department,you receive a notice
from the provincial / territorial medical regulatory authority that the family has made a formal complaint
about their relative's care.Which one of the following isyour best defense against allegations of negligent
care?

You did the best you could under the circumstances.


You acted as any reasonable physician in your area would have acted.
The coroner was notified and had no concerns about the death.
Your medical records clearly document all care provided.
The patient's previous medical history made death almost inevitable.

Question: 17

Which of the following would contribute the most to reducing mortality from cervical cancer in Canada?

Basic training in colposcopy for family physicians in rural and remote areas
Wider use of nurse practitioners in Papanicolaou smear testing
Better quality control in interpreting cytological specimens
Population-based Papanicolaou smear invitation letters and outreach programs
Papanicolaou smear recall programs for family practices

Question: 18

Cyclic itching or burning of the vulvar skin during the pre-menstrual phase of the cycle (cyclic vulvitis) is
often caused by

Candida albicans.
Bacterial vaginosis.
Trichomonas vaginalis.
Group B streptococcus.
Escherichia coli.
Question: 19

The skin lesion shown in the referenced illustration was the presenting complaint of a 35-year-old man.
Approximately 1 month after its appearance he developed pneumonia. Which one of the following
organisms is the most likely to have caused the pneumonia?

Streptococcus pneumoniae.
Legionella pneumophila.
Mycobacterium
tuberculosis.
Mycobacterium avium.
Pneumocystis carinii.

Question: 20

Efficacy of the results from a clinical trial of an antibiotic refers to:

the effects observed in a research setting.


the effects observed per unit cost of the drug.
the effects observed in a practice setting.
the effects observed adjusting for patient compliance.
the effects observed adjusting for physician compliance.

Question: 21

You diagnose acquired immune deficiency syndrome (AIDS) in one of your patients. Which one of the
following statements applies?

Reporting to health authorities is mandatory.


Reporting the patient's contacts is mandatory.
Consent of the patient is mandatory before reporting to health authorities.
No reporting is necessary because public health authorities provide periodic follow-up of patients
infected with the human immunodeficiency virus.
No reporting is necessary if previously diagnosed human immunodeficiency virus infection was
reported.
Question: 22

A 25-year-old woman presents to the Emergency Department complaining of a 1-week history of blurry
vision of her left eye and a dull eye ache worse on eye movement. On physical examination of the left eye,
you note 20/40 vision, decreased color perception, a central scotoma, and an afferent pupillary defect. The
right eye shows 20/20 vision and is otherwise normal. Which one of the following investigations is most
appropriate at this time?

Slit-lamp examination
Magnetic resonance imaging (MRI) of the brain
Anti-nuclear antibodies (ANA)
Tensilon (edrophonium) challenge
Acetylcholinesterase receptor antibodies

Question: 23

Which one of the following is the most useful intervention in preterm labor at 27 weeks' gestation?

Betamethasone injections
Magnesium sulphate bolus and then infusion
Intravenous hydration
Bed-rest
Erythromycin orally for 1 week

Question: 24

A 27-year-old married woman with a 5-year-old daughter discloses a history of verbal and physical abuse
of several years' duration. Her husband has never hurt her daughter, but she fears for her own safety. When
counseling her which one of the following is the most appropriate?

When anticipating an argument or beating, avoid areas without access to an outside door.
If the situation is serious, avoid "giving in" to the partner as this will only make the situation
worse the next time.
Do not disclose the abuse to neighbors as they may call the police when they hear suspicious
noises from her house.
Avoid talking to her daughter about the violence
Hide a car key in the basement so that she can leave quickly.
Section: 2

Question: 1

A 28-year-old gravida 2, aborta 1 at 13 weeks' gestation is Rh negative and, anti-D antibodies are identified.
What is the appropriate next step?

Initiate serial obstetric ultrasound assessments.


Assess the antibody titer.
Reassess her in 4 weeks.
Give her 300 units of Rh immune globulin intramuscularly.
Refer her for serial amniocentesis.

Question: 2

A 10-year-old girl presents with a 1-month history of intermittent fevers, weight


loss and abdominal pain. Her stool is positive for occult blood and her
erythrocyte sedimentation rate is 50 mm/hour. Examination reveals mouth
ulcers. These findings are most suggestive of which one of the following
diagnoses?

Pseudomembranous colitis.
Gluten enteropathy.
Crohn disease.
Eosinophilic gastroenteritis.
Salmonellosis.

Question: 3

A 27-year-old woman suffers from end-stage muscular dystrophy and has asked
you to discontinue respiratory support. She understands that her breathing
depends on the ventilator. She is tearful when making her request. A mental
status examination shows her cognitive functions to be normal and no
depression is diagnosed. Her spouse opposes suicide and euthanasia. Which one
of the following should you do?

Take appropriate steps to carry out her request.


Disregard her wish because it represents euthanasia.
Obtain consent from the spouse.
Refer her for a second psychiatric opinion.
Disregard her wish because it represents assisted suicide.
Question: 4

Which one of the following is the most useful finding in differentiating a


manic episode from a psychosis associated with schizophrenia?

Hostile behavior.
Excitement.
Anxiety.
Grandiose delusions.
Thought broadcasting.

Question: 5

A 65-year-old woman presents because she has noticed a mass in her


neck. Examination reveals a non-tender, fixed, 1 cm mass in the left lobe of the
thyroid gland. Which one of the following features would prompt you to refer
her for a biopsy?

Previous irradiation of the neck.


Known multi-nodular goiter.
Prior treatment with radioactive iodine.
Previous treatment with propylthiouracil.
Hot nodule on thyroid radioisotope scan.

Question: 6

A father brings his 9-month-old boy to your office with a concern of poor
weight gain. The infant has no other symptoms. His physical examination is
unremarkable. Referencing the attached growth chart (next page), which one of
the following would be the next best step?

Reassure the father and follow up in 3 months


Send patient for urgent computerized tomography of the head
Ask the father to keep a detailed diary of feedings and stools
Arrange for admission to hospital
Call the children's protective services
Question: 7

A four-year-old girl is brought to you for assessment of intoeing. She trips occasionally but is normally active
for her age. She has no complaints of pain. On physical examination, she has increased femoral anteversion.
Her tibial torsion and foot exam are normal. Which one of the following is the most appropriate management?

Derotation osteotomy of the femurs.


Positional exercises.
Night splints.
Observation.
Physiotherapy.

Question: 8

A 31-year-old woman consults you for the insertion of an intra-uterine device. She has no concomitant disease
and takes no medication. The gynecological examination is normal and all screening tests were negative.
While sounding the uterus, you inadvertently puncture the uterine fundus. The patient notices some pain but
tolerates it well and the rest of the procedure goes well. You should

tell her that the insertion was technically more difficult than usual and discharge her for close follow-
up.
tell her that you were negligent during the procedure and offer your support for any future actions.
avoid telling her that her uterus was punctured as there is no consequence.
tell her that her uterus was punctured but the risk of complications is small and arrange for follow-up.
avoid telling her that her uterus was punctured but notify your insurance provider of the incident.

Question: 9

A 60-year-old woman wishes to discuss stopping her estrogen replacement therapy. Which one of the
following would be the best reason to continue her present regimen of low-dose estrogen?

A personal history of heart disease.


A family history of myocardial infarction.
Persistent hot flashes and insomnia when off hormones.
A family history of osteoporosis.
A family history of cerebrovascular accidents.
Question: 10

A 65-year-old man has a right hemicolectomy for cancer. On post operative day 2, he has a temperature of
38.2 C. Which one of the following is the most likely diagnosis?

Urinary tract infection


Wound infection
Atelectasis
Deep vein thrombosis
Pneumonia

Question: 11

A 40-year-old woman comes to you with menstrual problems. Her periods are coming more frequently, and
she cannot predict when her period will start. They have been so heavy at times that she has leaked through her
clothes. Hemoglobin is 118 g/L. Which one of the following blood tests would be most useful in helping you
decide on treatment?

Serum iron.
Serum progesterone.
Ferritin.
Follicle-stimulating hormone (FSH).
Thyroid-stimulating hormone (TSH).

Question: 12

A patient is brought to the hospital by his friends following drug abuse. He is hallucinating, suspicious, and
violent. His gait is unsteady, and he is repeatedly making involuntary grimaces. Which one of the following
agents is most likely the cause of this disorder?

LSD.
Heroin.
Phencyclidine.
Mescaline.
Volatile solvent.
Question: 13

A 70-year-old British engineer presents with a 3-month history of general malaise, tiredness and memory
problems. During the Folstein's Mini-Mental State Examination (MMSE), he looks tense but remains alert. He
loses 2 points on spontaneous recall, retrieving the missed items with categorical prompting and 2 points on
spelling "world" backwards. He refuses to try "serial sevens" stating he is too tired. His clock face drawing is
accurate. Which one of the following is the most likely diagnosis?

Probable Alzheimer dementia.


Multi-infarct dementia.
Depression.
Frontal lobe dementia.
Delirium.

Question: 14

A 60-year-old woman has been booked for a laparoscopic cholecystectomy for symptomatic gallstones. Prior
to her surgery, she presents to the Emergency Department with a history of feeling faint and unwell. She has
had rigors. On physical examination, her temperature is 40C. Her white blood count is 22 x 109/L; aspartate
aminotransferase, 63 U/L; alanine aminotransferase, 78 U/L; alkaline phosphatase, 450 U/L; amylase level,
200 U/L and bilirubin, 50 mol/L. Which one of the following is the most likely diagnosis?

Cholecystitis.
Pancreatitis.
Cholangitis.
Duodenal ulcer.
Hepatic abscess.

Question: 15

A 45-year-old woman complains of bright red postcoital bleeding which has been occurring for the last 2
months each time she has intercourse. On pelvic examination, the cervical contour is irregular, and there is a 2
cm ulcerative lesion on the anterior lip. Which one of the following is the most likely diagnosis?

Cervical syphilis.
Cervical carcinoma in situ.
Herpes cervicitis.
Chlamydial cervicitis.
Cervical carcinoma.
Question: 16

A 60-year-old woman presents to your office complaining of increasing fatigue. She has been treated with
metoprolol for the last 6 months for high blood pressure. She has fibromyalgia and has recently started a trial
of amitriptyline to get better sleep. She denies weight loss, palpitation and sleep disorders. Her physical
examination is otherwise completely normal. Which one of the following is the most likely cause?

Depression.
Occult malignancy.
Medication side effects.
Hypothyroidism.
Chronic fatigue syndrome.

Question: 17

A 55-year-old woman has had a 5-year history of daily crampy abdominal pain in the left lower quadrant.
Bowel movements tend to vary between loose and hard stools. Her weight has been stable and she has not
noted any blood in her stool. Physical examination reveals some pain on palpation of the left lower quadrant.
Rectal examination is normal. A double contrast barium enema reveals diverticulosis of the sigmoid colon.
Proctoscopy is normal. Which one of the following is the most appropriate treatment?

Dietary counseling.
Sigmoid resection.
Oral steroids.
Two-week trial of oral antibiotics.
Antidepressive medication.

Question: 18

A 57-year-old man with 2 previous myocardial infarctions is brought to the Emergency


Department unresponsive. His blood pressure is not detectable. The electrocardiogram demonstrates
ventricular fibrillation. He is intubated and cardiopulmonary resuscitation is initiated. He is placed on a
monitor and intravenous access is obtained. He is defibrillated at 200, 300, and 360 joules and remains in
ventricular fibrillation. Which one of the following is the most appropriate next step?

Defibrillate again at 360 joules.


Administer 1.5 mg/kg of intravenous lidocaine.
Administer 1 mg of intravenous epinephrine.
Administer 1 mg of intravenous atropine.
Perform pericardiocentesis to rule out cardiac tamponade.
Question: 19

A 40-year-old man presents with abdominal pain. He is known to have some antisocial traits. He states that he
requires a certificate to excuse himself from work. Your examination is negative. When confronted, he admits
to pressure at work. The most likely diagnosis is

factitious disorder.
hypochondriasis.
generalized anxiety disorder.
conversion disorder.
malingering.

Question: 20

A primigravida patient, following a prolonged first stage of labor, has been in the second stage of labor for 3
hours with arrest of descent and rotation. The caput succedaneum is at station 0 in an occipitoposterior (OP)
position. Which one of the following management options is the most appropriate?

Vacuum extraction delivery.


Simpson forceps and delivery in OP position.
Kjelland forceps rotation and delivery.
Manual rotation and forceps delivery.
Lower segment cesarean section.

Question: 21

A 63-year-old gravida IV, para II presents with a 2-year history of a sensation that something is ""bulging""
out of her vagina during straining. She is not emptying her bladder completely and often ""double voids."" She
has no urinary incontinence. She has no gastrointestinal symptoms. Past gynecologic history is unremarkable
apart from an abdominal hysterectomy and oophorectomy. The most likely diagnosis is

enterocele.
rectocele.
procidentia.
detrusor dyssynergia.
cystocele.
Question: 22

In Canadian hospitals, which one of the following is customarily one of the obligations attached to the
privileges accorded to active staff physicians?

Participating in medical administrative committees.


Creating the duty roster for on-call coverage.
Voting on disciplinary measures for fellow staff members.
Approving nominations of new medical staff.
Evaluating the quality of nursing care if head of service.

Question: 23

A 38-year-old man has been in a persistent vegetative state for 10 years. The treating physician chose to
continue active treatment in spite of the patient's clearly expressed wishes to the contrary, stated in the chart.
The resident is called to assess an acute fever in this patient and feels that she should respect the patient's
autonomy and not treat the fever. What is the most appropriate initial course of action?

Discuss the apparent conflict with the attending physician.


Call the provincial/territorial licensing authority.
Call the medical protective association.
Request an ethics consultation.
Suggest to the chief of staff/medical director of the hospital that the patient is getting inappropriate
care.

Question: 24

You have been subpoenaed to produce a patient's chart. Which one of the following measures is most
appropriate? The physician:

must produce only the paper record.


may withhold any private notes he/she might have.
must produce only a summary of the record.
must not produce any records unless the patient consents to this.
must produce the complete original paper record plus any pertinent electronic records.
Section: 3

Question: 1

The laws of Canada relating to informed consent require that patients making decisions about their health care
be offered information on the proposed treatment. Material risks that must be disclosed to patients are

risks they should know about.


risks they would want to know about.
risks that are likely to cause harm.
unlikely risks that may cause harm.
risks of undergoing or foregoing the treatment.

Question: 2

A 26-year-old man from the local military base presents to your office complaining of severe pain over the
tailbone. Physical examination reveals a swollen, indurated area over the natal cleft. Which one of the
following is the most appropriate management at this time?

Incision and drainage.


Oral antibiotics.
Oral analgesics and rest.
X-ray of the sacrum.
Injection of steroids.

Question: 3

A mother consults you in the Emergency Department because her 3-week-old breast-fed baby has had
significant crying episodes that last 2 or 3 hours a day, during which he is difficult to console. The stools are
normal in appearance but bowel movements have only occurred every 2 to 3 days since birth. The baby is
feeding normally, and the physical examination is normal. Which one of the following courses of action is the
most appropriate?

Give advice to the mother on relieving colic.


Request a sweat chloride test.
Refer the baby for a rectal biopsy.
Request an abdominal ultrasound.
Modify the baby's diet to a soy milk-based formula.
Question: 4

A 65-year-old man presents with rectal bleeding. On proctoscopic examination, you identify an internal
hemorrhoid that bleeds but does not prolapse. Which one of the following is the best treatment?

Stool softener
Laser
Anal dilation
Hemorrhoidectomy
Cryotherapy

Question: 5

You are invited to participate in the planning of a program to increase physical activity in the community.
What is the first information you need to know about this community?

The current level of physical activity.


The incidence of heart disease.
The number of fitness facilities.
The level of obesity.
Wealth.

Question: 6

A teenager who underwent appendectomy 1 week ago for acute appendicitis with perforation now has a fever.
The chest x-ray shows a right pleural effusion. Which one of the following tests is the most cost-effective to
confirm your suspicion of a subphrenic abscess?

Three plain view x-rays of the abdomen.


Liver-lung nuclear medicine scans.
Ultrasound scan of the abdomen.
Lateral view chest x-ray.
Computed tomography scan of the abdomen.
Question: 7

In a trial of 1000 children with acute Kawasaki disease, high dose intravenous gamma globulin (IVIG) was
compared to low dose IVIG. The risk of developing a coronary aneurysm was 3.2% in the low dose IVIG
group and 1.2% in the high dose IVIG group. How many children would you need to treat with high dose
IVIG in order to prevent 1 child from having a coronary aneurysm?

20
100
200
50
25

Question: 8

A 45-year-old alcoholic is brought to the Emergency Department after a fall in a bar. He drank approximately
twelve beers in 3 hours. A few hours later, he experienced a seizure on the right side of the body. Which one of
the following diagnoses is most likely?

Hypoglycemia.
Hepatic encephalopathy.
Subdural hematoma.
Withdrawal seizure.
Wernicke encephalopathy.

Question: 9

A 26-year-old woman presents to the Emergency Department with complaints of recurrent episodes of
dyspnea and cough despite use of salbutamol by inhalation. She appears anxious and tremulous. Examination
reveals respiratory rate of 24/minute, blood pressure 136/76 mm Hg, pulse 118/minute, forced expiratory
volume and peak flow 80% of predicted, and a low-pitched expiratory wheeze best heard over upper chest and
neck. Which one of the following measures is the most appropriate management?

Oxygen and IV aminophylline.


Salbutamol by mask, IV valium.
Decrease salbutamol, give inhaled steroids.
Subcutaneous epinephrine, sublingual lorazepam.
Decrease salbutamol, give oral aminophylline.
Question: 10

A 55-year-old woman presents with a long history of indigestion and belching after fatty meals. Gallstones
were found on ultrasound. Her gallbladder was subsequently resected and an operative cholangiogram was
negative. She was free of the symptoms for several weeks but they then recurred. Which one of the following
is the most likely explanation for the recurrence of the symptoms?

A few stones were left in the common duct.


Pancreatitis has resulted from the cholecystectomy.
The removal of the gallbladder has caused impairment of fat digestion.
The patient has cystic duct stump syndrome.
The patient's original symptoms were unrelated to gallstones.

Question: 11

This 4-day-old infant was delivered at term to a healthy mother following a prolonged labor. On the third day,
the baby appears to be doing well but the nurse draws your attention to the appearance of the infant as shown
in the referenced illustration. What should be your most appropriate action?

Order coagulation studies followed by


administration of Vitamin K.
Reassure the mother that there is no reason for
concern.
Inquire about maternal ingestion of
diethylstilbestrol.
Order a pelvic and abdomen ultrasound.
Consult with a gynecologist.
Question: 12

A 3-year-old girl is seen in the Emergency Department with high fever, redness of the right eye and proptosis
with limitation of extraocular movements. Which one of the following is the most likely diagnosis?

Dacryocystitis.
Maxillary sinusitis.
Periorbital cellulitis.
Orbital cellulitis.
Blow-out fracture of the orbit.

Question: 13

A 32-year-old gravida II, para II presents to your office. On speculum examination, you visualize a smooth 1-
cm, yellow, translucent area adjacent to the external os. This most likely represents

an endometrioma.
a Nabothian cyst.
a squamous carcinoma of the cervix.
a cervical condyloma.
a cervical polyp.

Question: 14

The highest prevalence of schizophrenia is found in which one of the following specific populations?

Child with one schizophrenic parent.


Non twin sibling of schizophrenic patient.
Digygotic twin of schizophrenic patient.
Adopted child of a schizophrenic mother.
Monozygotic twin of schizophrenic patient.
Question: 15

You are asked to assess a 47-year-old man in the Emergency Department with a 2-day history of
severe epigastric pain radiating to his back, nausea and vomiting. He admits to excessive alcohol use in the
past. On examination, his abdomen is slightly bloated, very tender, and he has reduced bowel sounds. His
white cell count is 18,000, serum lipase 840, and his serum calcium is 1.80. His blood pressure is 80/40
mmHg, heart rate 120/minute, and oxygen saturation is 78%. What is the most likely cause of these findings?

Loss of fluid into the retroperitoneal space.


Sepsis resulting from Gram-negative bacterial growth in necrotic pancreatic tissue.
Loss of whole blood into the peritoneal cavity.
Deleterious effects of trypsin and other toxic products on cardiac function.
Disseminated intravascular coagulation.

Question: 16

A distraught mother brings her five-year-old son to your office. She states that she found her son and his six-
year-old friend in the bedroom with their pants down. The mother states that a relative had sexually abused her
in the past. She wants you to examine her son to make sure he is all right. Which of the following would you
do during the course of the interview?

Arrange an interview with the six-year-old and parent.


Notify child protection services.
Reassure the mother and arrange counseling sessions for the boy.
Separate mother and child to further explore mother's concerns.
Ensure the children have better supervision when playing.

Question: 17

A 75-year-old man presents with abdominal pain, associated with a tender mass at the umbilicus, which
appeared 24 hours earlier. He has also been experiencing nausea and vomiting. On examination, he has tender
mass at the umbilicus, which is reddened. He is also distended but has no peritoneal findings. An x-ray (next
page) shows air fluid levels, and dilated loops of small bowel. Along with nasogastric drainage, and
intravenous fluids, which one of the following is most appropriate at this time?

Reduction of the hernia


Fine needle biopsy of the mass
Laparotomy
Incision and drainage of the mass
Observation
Question: 18

A 6-year-old asymptomatic child presents with a first episode of hematemesis. On physical examination, she
has no spider nevi, ascites or jaundice. Her spleen is palpable 4 cm below the costal margin and her liver span
is normal. Heart rate is 140/minute and blood pressure is 74/45 mm Hg. Family history is negative for liver
disease. Which one of the following is the most appropriate management of this upper gastrointestinal
bleeding?

Intravenous use of an H2 blocker.


Oral use of an H2 blocker.
Gastroscopy and sclerotherapy of the varices.
Gastroscopy and laser cauterization of the bleeding ulcer.
Intermittent nasogastric suction and intravenous volume replacement.

Question: 19

A 40-year-old man presents to the Emergency Department stating that his psychiatrist has recently increased
his medication. He cannot sit or stand for long and has to keep pacing. Which one of the following diagnoses
is most likely?

Acute dystonia.
Anxiety.
Acute akathisia.
Panic reaction.
Tardive Dyskinesia.
Question: 20

A 75-year-old man has had recurrent episodes of hemoptysis and right middle lobe infiltrates in the last year.
Which one of the following diagnoses is the most likely?

Recurrent pulmonary emboli.


Bronchiectasis.
Aspiration pneumonitis.
Lung abscess.
Chronic bronchitis.

Question: 21

A 24-year-old man presented with a mass in his left groin. It appeared suddenly 2 hours ago while lifting a
piano. On examination, he has a tender, firm mass in the left groin.Which one of the following is the next best
step?

Immediate hernia repair


Ice packs to groin
Needle aspiration
Ultrasound of groin
Reduction of mass

Question: 22

A 32-year-old woman comes to your office for her 6-week postpartum examination. She complains of fatigue
and palpitations. Thyroid examination reveals a small nodule. Laboratory investigation confirms your
suspicion of postpartum thyroiditis with thyrotoxicosis. Which one of the following is the most appropriate
management?

Methimazole
Propylthiouracil
Thyroid hormone
I131 therapy
Beta-adrenergic blocker
Question: 23

An obese 45-year-old man suddenly develops nausea and abdominal pain associated with a tender swelling in
the left groin. Physical examination shows a distended abdomen with an irreducible mass in the left groin.
Plain abdominal x-rays show distended bowel loops. In addition to intravenous fluids and nasogastric suction,
which one of the following is the most appropriate management?

Needle aspiration of the mass.


Immediate surgery.
Bed rest and analgesia.
Application of a truss.
Intravenous antibiotics.

Question: 24

A 76-year-old man presents to the Emergency Department 8 hours after the sudden onset of right arm
weakness and problems with word finding. Past medical history is positive for type II diabetes mellitus.
Medications include acetylsalicylic acid, glyburide, and metformin. On examination, his blood pressure is
180/100 mm Hg, heart rate 60 and regular. Neurological examination confirms an expressive aphasia and
upper motor neuron weakness in the right arm. Computerized tomography scan of the head is negative. Which
one of the following treatments has been demonstrated to reduce the likelihood of a second cerebrovascular
accident?

Intravenous labetalol
Thrombolytic therapy with tpa
Aggressive diabetes management
Dipyridamole/ASA (Aggrenox)
Intravenous heparin
Section: 4

Question: 1

A 5-year-old boy is brought to the Emergency Department by ambulance after being rescued from icy cold
ocean water 15 minutes ago. He fell from a wharf and was submerged for approximately 5 minutes. On
arrival, he is in cardiac arrest, and his core temperature is 30.4C. Which one of the following is the
appropriate early management of this patient?

Warm the patient before starting cardiopulmonary resuscitation (CPR).


Pronounce death secondary to drowning.
Continue CPR throughout the warming phase.
Continue CPR at half the recommended rate.
Continue CPR at double the recommended rate.

Question: 2

A 75-year-old woman is admitted to hospital for hip fracture following a fall while intoxicated. She develops
excessive worries, undue pessimism, poor appetite, and early morning awakening. She is having difficulty
with physiotherapy due to poor energy and excessive pain. She is treated with sertraline 150 mg daily to which
she responded in the past. Which one of the following is most likely to increase her risk of suicide?

Excessive worrying
Poorly controlled pain
Abstinence from alcohol consumption
Early morning wakening
Female gender

Question: 3

A 14-year-old boy presents to the Emergency Department after colliding with a telephone pole while riding his
bicycle. He was not wearing a helmet. His vitals signs are stable. He is alert and oriented and smells of
alcohol. His mother reports that he was involved in a similar incident 7 months ago. Which one of the
following should be performed prior to discharge?

Notify school guidance counselor.


Social work consultation.
Counsel patient to attend Alcoholics Anonymous.
Notify law enforcement agency.
Start an antidepressant.
Question: 4

A 16-year-old female consults in the Emergency ward for severe vulvar burning that increased rapidly over the
last three days. The vulvar appearance is shown on the illustration. What is the most likely diagnosis?

Condyloma accuminatum.
Candidiasis.
Herpes genitalis.
Contact dermatitis.
Behet disease.

Question: 5

A 30-year-old primigravida delivered a 4500 g male infant vaginally and subsequently had a severe
postpartum hemorrhage due to placental retention. The patient attempted to breastfeed the infant, but this
failed. During an office visit at 20 weeks' postpartum, she complains of severe fatigue and failure to resume
menstruation. Which one of the following is the most likely diagnosis?

Asherman syndrome
Post partum thyroiditis
Panhypopituitarism
Grave's disease
Post-partum depression
Question: 6

You are asked to see a 78-year-old woman who has become confused and irritable. She has been fed with a
gastrostomy tube for 2 months because an esophageal carcinoma has prevented her from eating or drinking.
She is hypotensive and your examination suggests significant dehydration. Her electrolytes show the
following results:

Sodium 165 mmol/L


Potassium 4.8 mmol/L
Chloride 115 mmol/L
Bicarbonate 24 mmol/L

Which one of the following is the most appropriate initial intravenous fluid?

5% dextrose/water.
10% albumin.
0.45% saline.
3.3% dextrose/0.3% saline.
0.9% saline.

Question: 7

A 22-year-old female with known Crohn's disease of the small bowel presents with clinical and radiological
signs of obstruction. After placement of a nasogastric tube, which one of the following would be the best
initial management plan?

Total parenteral nutrition.


Small bowel resection.
IV steroids.
Steroid enema.
Colonoscopy with dilation of stricture.
Question: 8

A 32-year-old gravida 2, para 1 presents at 10 weeks' gestation for prenatal counseling. Her first child has
Down syndrome, and she wants to know if this baby is affected. Which one of the following is diagnostic?

Amniocentesis
Maternal serum screen at 16-18 weeks
Maternal serum screen at 11-13 weeks
Parental karyotype
Nuchal translucency at 11-14 weeks

Question: 9

A 10-year-old girl has been taking methylphenidate for attention deficit disorder. She has not responded to an
adequate dose of her medication. The most appropriate alternative medication would be

imipramine.
fluoxetine.
desipramine.
monoamine oxidase inhibitor.
dexamphetamine.

Question: 10

A previously healthy 2-year-old boy presents to your walk-in clinic with a 3-day history ofvomiting and a 2-
day history of watery diarrhea. His physical examination reveals a heart rate of 100/minute, a blood pressure of
100/60 mmHg, and a respiratory rate of 24/minute. His mouth is dry, his eyes are not sunken, and his skin
turgor is normal. The rest of his examination is completely normal. Which one of the following would be most
appropriate for this patient?

Loperamide.
Intramuscular dimenhydrinate.
Intravenous rehydration.
No medication.
Bismuth salicylate.
Question: 11

A 62-year-old male smoker is found at an insurance exam to have a Hgb. of 185 and Hct of 0.59, WBC 4.95.
What is the most likely explanation for his abnormal results?

Hemochromatosis
Volume depletion
Renal cell carcinoma
Polycythemia rubra vera
Chronic pulmonary disease

Question: 12

A 55-year-old man and his wife present to your office and state that they are having marital problems because
the husband has erectile dysfunction. The problem has been getting worse for the last 2 years, and there has
been no sexual intercourse for 6 months. The husband tells you privately that he still has spontaneous erections
when he wakes in the morning and when he reads sexually explicit magazines. He has hypertension and has
been taking propranolol for the past 5 years. Which one of the following is the most appropriate course of
action?

Monitor nocturnal penile tumescence.


Stop the propranolol.
Initiate marital therapy without further physical investigation.
Measure the serum testosterone level.
Request a glucose tolerance test.

Question: 13

A 74-year-old woman presents to the office with a 4-month complaint of fatigue, muscle weakness and mild
involuntary weight loss. Stool frequency is increased but there is no change in its character. She has been
taking conjugated estrogen (Premarin) 0.625 for 18 years. Laboratory investigations show a normal
hemoglobin and urinalysis, blood pressure 148/78 mm Hg, pulse 80/minute and irregularly irregular. Which
one of the following laboratory test results best explains this scenario?

Serum potassium3.0 mmol/L.


Thyroid-stimulating hormone 0.01 mU/L.
Serum calcium2.4 mmol/L.
Serum creatine kinase 640 U/L.
Hemoglobin 110 g/L.
Question: 14

A patient complains that information written in her hospital chart by her attending physician about a previous
suicide attempt is harmful to her because it causes staff to mistreat her. In general, a physician's statement in a
hospital chart will not be found to be libelous unless it

is untrue.
does not serve the public interest.
harms the patient's reputation.
represents a breach of confidentiality.
was motivated by malicious intent.

Question: 15

An 83-year-old woman suffers from a probable Alzheimer-type dementia. Usually, by the end of the
afternoon, she shows recurrent psychomotor agitation. She insists upon leaving the institution claiming her
parents are waiting for her at home. Behavioral interventions do not calm her. When supper comes, the patient
often refuses to eat, endlessly waiting for her father to pick her up. After numerous trials, her physician plans
to prescribe neuroleptics. Which one of the following statements is correct?

The use of neuroleptics is ethically acceptable only after a precise psychiatric diagnostic is
established.
Even if neuroleptic malignant syndrome is a rare side effect (< 1%), it must be divulged for consent
to be valid.
The patient herself must first give consent.
Late dyskinesia will only be mentioned amongst side effects if drug therapy calls for prolonged use
of neuroleptics.
Chemical restraint by use of neuroleptics can only be approved by a psychiatrist.

Question: 16

You are called to the intensive care unit to see a patient for anterior myocardial infarction. The admission
electrocardiogram shows a left anterior hemiblock and a right bundle branch block. The patient is awake, pulse
is 32/minute and blood pressure is 95/70 mm Hg. Which one of the following is the best treatment?

Intravenous atropine.
Cardioversion.
Pacemaker.
Dopamine.
Isoproterenol.
Question: 17

The main modifiable determinant of childhood obesity is

sedentary behavior.
parental obesity.
consumption of junk food.
low self-esteem.
type 2 diabetes mellitus.

Question: 18

You examine a 5-year-old boy with a 3-week history of limping with pain in the right hip and knee. An x-ray
of the right hip reveals flattening of the femoral head with bone remodeling and widening of the hip joint
space. What other diagnostic test would be most appropriate to order initially?

Magnetic resonance of the right hip.


Joint aspiration.
Plain x-rays of the left hip.
Scoliosis x-ray series.
Complete blood count and erythrocyte sedimentation rate.

Question: 19

Sympathomimetic drugs may be used in the treatment of which one of the following conditions?

Hypomania.
Insomnia.
Attention deficit hyperactivity disorder.
Neuroleptic malignant syndrome.
Cocaine abuse.
Question: 20

A child, who has previously had a splenectomy, is brought to the Emergency Department with a fever of
39.7C. Which one of the following steps should be taken?

Give intravenous immunoglobulin.


Give intravenous antibiotics.
Give oral acetaminophen and cefaclor.
Do a complete blood count and observe for 6 hours.
Do a blood culture and follow up as outpatient.

Question: 21

In differentiating pseudodementia from true dementia, it is most important to

obtain an occupational history.


review medications thoroughly.
determine if the onset is gradual.
perform a thorough mental status examination.
consider a therapeutic trial of benzodiazepines.

Question: 22

A 40-year-old man with no previous history of cardiac disease presents to hospital due to palpitations and
lightheadedness. On initial examination, he is alert and oriented. His blood pressure is 90/60 mm Hg and heart
rate, 180/minute. His electrocardiogram shows a wide complex tachycardia. Which one of the following
findings most strongly suggests that this man's arrhythmia is supraventricular in origin?

Delta wave on premorbid electrocardiogram.


Presence of fusion beats on electrocardiogram.
Presence of cannon A waves on examination of the jugular venous pressure.
Lack of transition of QRS complexes from V1 to V6.
Presence of capture beats on electrocardiogram.
Question: 23

A 30-year-old man presents with irritability and insomnia. He impulsively quit his job after arguing with his
boss. Although he has limited experience, he is confident that he will succeed if he starts his own company. He
has a history of major depression and alcohol abuse. Which one of the following is the most appropriate long
term pharmacological treatment for this man?

Clonazepam
Lithium carbonate
Flupenthixol
Gabapentin
Fluoxetine

Question: 24

Clinical office examination of a 5-day-old infant includes auscultation of the chest where you hear a
widespread pansystolic heart murmur which is best heard over the infant's left scapula. The infant is otherwise
asymptomatic. Further examination reveals an unremarkable abdominal examination except for weak femoral
pulses. The next step in managment is which one of the following?

Reassess this infant in 1 week's time.


Book an elective chest radiograph closer to the 1 month visit.
Further investigation if the murmur is present on the 1 month clinical examination.
Reassure the mother that antenatal ultrasounds showed no cardiac anomalies.
Order an urgent echocardiogram.
Self-Administered QEI-MCQ Exam 2
Section: 1

Question: 1

A 64-year-old man was diagnosed with atrial fibrillation and mild congestive heart failure 3 weeks ago and, at
that time, was started on warfarin 4 mg per day and digoxin 0.25 mg per day. Today, he has no evidence of
heart failure, his pulse is 80/minute and irregular, and his International Normalized Ratio is 1.84. Which one of
the following would be the most appropriate treatment at this time?

Increased warfarin.
Increased digoxin.
Decreased digoxin.
Cardioversion.
Decreased warfarin.

Question: 2

A 47-year-old man presents to your office complaining of persistent hoarseness, chronic mild dry cough, and
frequent clearing of his throat. He denies indigestion but does notice on occasion a bitter taste in his mouth in
the morning. Which one of the following is the most likely diagnosis?

Chronic nasopharyngitis
Laryngopharyngeal reflux disease
Reactive airways disease
Hiatus hernia
Multinodular goitre

Question: 3

A 38-year-old woman presents to the Emergency Department for the fourth time in a month with complaints of
sudden onset of chest pain, shortness of breath, palpitations, dizziness and numbness in her hands. On each
occasion, she had a complete medical evaluation with no organic findings to justify her symptoms. The most
appropriate pharmacological treatment for this patient is

risperidone daily for 6 months.


perphenazine twice a day as necessary.
buspirone three times a day as necessary.
perphenazine daily for 6 months.
paroxetine daily for 6 months.
Question: 4

An 11-year-old girl presents to the Emergency Departmentwith a history of recurrent heavy vaginal
bleeding.She is hemodynamically stable. Her hemoglobin is 105 g/L, andher coagulation studies are normal.
Which one of the following is your initial management?

Intramuscular iron therapy.


Dilation and curettage.
Blood transfusion.
Hormone administration.
Administration of fibrinogen.

Question: 5

A pharmacist reports "double doctoring" of a controlled drug by your patient. Which of the following actions
is the appropriate first step?

Report the patient to Health Canada, Office of Controlled Substances.


Ask the pharmacist to notify the police.
Notify all pharmacies and other clinics in the area.
Try to resolve the matter directly with the patient.
Report the patient to the police.

Question: 6

Which one of the following screening tests done in the first trimester of pregnancy may detect trisomy 21?

Maternal serum screen (alpha-fetoprotein, beta-HCG, estriol).


Increased alpha-fetoprotein.
Increased nuchal translucency on ultrasound.
Amniocentesis.
Decreased fetal heart rate.
Question: 7

A 20-year-old man is involved in a motor vehicle collision. He was the passenger, and he was wearing his seat
belt. His only complaint at presentation was vague abdominal pain at the site of a seat belt mark. He was
admitted for observation and overnight developed increasing abdominal pain, fever, guarding and rebound. His
vitals were stable, and an abdominal series showed a couple of air fluid levels, with minimally dilated small
bowel. Which one of the following is the most likely diagnosis?

Liver laceration
Abdominal wall hematoma
Ruptured spleen
Bladder rupture
Diaphragmatic rupture

Question: 8

A 65-year-old man was diagnosed with sigmoid diverticulitis on CT scan. He was treated successfully with IV
antibiotics. Following discharge from hospital, which one of the following is the most important test for him to
have now?

CBC.
Ultrasound.
Colonoscopy.
Repeat CT scan.
Rigid sigmoidoscopy.

Question: 9

A 16-year-old adolescent presents to your office and tells you that she was told by a cardiologist that she has
an atrial septal defect. On auscultation of her heart, which one of the following findings would you expect to
hear?

Paradoxically split second heart sound.


Fourth heart sound.
Loud first heart sound.
Third heart sound.
Fixed split second heart sound.
Question: 10

A 40-year-old man presents with several days of upper abdominal pain, associated with nausea and vomiting.
He has obvious peritonitis across his upper abdomen. His lab work shows an amylase of 1000U/L, aspartate
aminotransferase 250U/L, alanine aminotransferase 200U/L, alkaline phosphatase 190 U/L, total bilirubin
26umol/L, white blood cell count 17 x 109/L, and an x-ray as shown. Which one of the following is the most
likely diagnosis?

Choledocholithiasis
Acute cholecystitis
Perforated duodenal ulcer
Acute pancreatitis
Cholangitis

Question: 11

A 47-year-old man has been training to run a marathon. After a long run, he develops mild pain and swelling
in his left knee. Examination shows a mild joint effusion and a soft tissue mass in the popliteal fossa.
Ultrasound confirms that he has a Baker's cyst. Which one of the following is the best treatment for this
condition?

Arthroscopic debridement.
Cyst aspiration and compression.
Surgical excision of the cyst.
Rest, compression and anti-inflammatory medication.
Intra-articular steroids.
Question: 12

An 82-year-old woman presents with a mass protruding through the vagina. On examination, you confirm a
procidentia with complete eversion of the vagina. Which one of the following recommendations is
appropriate?

She should avoid pessaries because of systemic complications.


She should avoid surgery in view of her age.
Vaginal hysterectomy and repair is the appropriate permanent solution.
She may avoid the need for intervention by reducing the uterus manually.
Vaginal repair without hysterectomy is a preferable surgical solution in view of her age.

Question: 13

You have just received confirmation that a patient of yours has inoperable carcinoma of the colon, with the
expectation of a very low 5-year survival rate. She is feeling weak and tired but is otherwise alert. Prior to the
investigations to confirm the diagnosis, the patient's family has requested that she not be informed of the
outcome of the investigations because it is their cultural tradition that the family members only be informed.
What would be the most appropriate action in this case?

Tell your patient in a sympathetic and caring manner the diagnosis and prognosis.
Tell the family you appreciate their views but you are legally obligated to inform your patient of the
diagnosis.
Discuss the matter with your patient and determine her wishes relating to the disclosure of such
information and abide by her wishes.
Abide by the wishes of the family.
Do not tell your patient because it may upset her to such an extent that it will greatly decrease the
quality of her remaining life.

Question: 14

In the late summer, a 7-year-old boy presents with persistent rhinorrhea and itchy eyes of 3 weeks' duration.
On examination, the nasal mucosa is pale and edematous. Which one of the following is the best management?

Treat with oral antibiotics.


Treat with oral antihistamines.
Treat with nasal decongestants.
Search for foreign body.
Refer to an ear, nose and throat specialist.
Question: 15

You learn while reading the newspaper that a man has been convicted of sexually assaulting a woman who is
your patient based on her testimony as to his actions. As the psychiatrist of this woman, you know that she is
suffering from a mental condition that makes her a very unreliable witness due to the presence of delusional
thinking. Which one of the following should you do?

Seek the advice of the Canadian Medical Protective Association so that they can represent you.
Contact the newspaper and reveal your information.
Contact the defense lawyer for the accused man so that he can properly cross-examine your patient.
Maintain the confidentiality of your information.
Contact the accused man and tell him of your knowledge.

Question: 16

A 45-year-old woman was involved in a motor vehicle collision. She was not restrained. She complains of
severe hip pain, and her hip is held in flexion and adduction. What is her most likely injury?

Posterior hip dislocation.


Pelvic fracture.
Femoral neck fracture.
Sacro-iliac joint disruption.
Psoas muscle disruption.

Question: 17

Which one of the following is the most likely diagnosis to be associated with the electrocardiogram shown in
the referenced illustration?

Aortic insufficiency.
Ventricular septal defect.
Mitral stenosis.
Acute myocardial infarction.
Acute pericarditis.
Question: 18

An 11-year-old girl, who is a Jehovah's Witness, is in need of a bowel resection for ulcerative colitis. Her
hemoglobin is 53 g/L (normal value 120-160 g/L). The family is refusing to allow her any blood
products. Which one of the following is the most appropriate next step?

Arrange to make her a ward of the court and give her a blood transfusion.
Transfer her care to another hospital.
Book her for surgery without any blood transfusion.
Ignore the patient's family's wishes and give her a blood transfusion.
Treat her nonoperatively.

Question: 19

A 31-year-old woman washes her hands repeatedly in a diluted bleach solution before changing her newborn's
diapers. She has fears of contaminating the child. Although she acknowledges that this behavior is "crazy", she
cannot stop herself and requests help. What is the most appropriate initial management?

Refer her to a psychodynamic psychotherapist.


Begin treatment with a selective serotonin reuptake inhibitor.
Inform the Child Protection Agency.
Refer her to a baby/mother support group.
Tell her to stop using bleach and substitute warm soapy water.

Question: 20

A patient suffering from psychosis who has not been compliant with his medication, threatens harm to a
coworker. The physician considers the threat to be serious and imminent. Which one of the following is the
most appropriate initial action?

Counsel the patient not to act on his threats.


Inform the patient's intended victim.
Advise the patient about potential legal action.
Inform the patient's family.
Change oral medications to depot-injection.
Question: 21

A 7-year-old girl with cystic fibrosis presents to your office with an acute exacerbation of her cough. On
examination, she is afebrile and in no apparent respiratory distress. Which one of the following is the most
appropriate treatment for this girl?

Intravenous immunoglobulin
Oral prednisone
Broad spectrum antibiotic
Oral anti-tussive (dextromethorphan)
Reassurance and book follow-up

Question: 22

A healthy nine-month-old child is in for a routine visit. On examination, you note the following findings (see
image.) Which one of the following conditions is this child most likely to have?

Pseudostrabismus.
Right esophoria.
Left esotropia.
Left coloboma.
Right congenital cataract.

Question: 23

A 50-year-old woman presents to the office with a complaint of tinnitus. On examination, she is found to have
a unilateral neural hearing loss. Which one of the following is the most likely cause of these symptoms?

Presbycusis.
Acoustic neuroma.
Cholesteatoma.
Otosclerosis.
Mnire disease.
Question: 24

A 39-year-old woman presents to your office and requests the birth control pill. She has used oral
contraceptives in the past with no problems. She is healthy and does not take any medications. Her mother had
breast cancer. Before prescribing the pill, which one of the following interventions would be most appropriate?

Assess her smoking status.


Review her past Papanicolaou smears for abnormalities.
Order a mammogram.
Do a urine pregnancy test.
Draw blood for a lipid profile.

Section: 2

Question: 1

A 65-year-old woman consults you because of mild indigestion. You discover a non-tender firm smooth mass
which moves with respiration in the right upper abdomen. Which one of the following is the best initial
investigation?

Gastroscopy.
Ultrasound scan.
Upper gastrointestinal series x-rays.
Tc-99 liver scan.
Plain x-rays (upright and supine) of the abdomen.

Question: 2

A 2-month-old child is brought in by her single adolescent street parent. You are concerned that this child may
not be gaining weight appropriately. Which one of the following factors would best indicate possible infant
malnutrition?

Weight gain of less than 1 kg over birth weight.


Lack of solid foods.
Lack of multivitamin supplementation.
History of cigarette smoking during gestation.
Head circumference growth of 5 cm.
Question: 3

A 21-year-old woman has had persistent superficial dyspareunia forthe last year. She is otherwise
asymptomatic. Vaginal cultures have been negative on 3 occasions. On examination, the skin located just
exterior to the hymen is slightly reddened and painful to the touch. Which one of the following is the most
likely diagnosis?

Allergic vulvitis.
Lichen sclerosus.
Human papillomavirus infection.
Vestibulitis.
Persistent herpes infection.

Question: 4

A 5-year-old boy with leukemia develops high fever, urticaria, and headache while receiving platelet
concentrates. Which one of the following measures is most appropriate in the initial management of this
problem?

Give acetaminophen.
Give subcutaneous epinephrine.
Stop the platelet transfusion.
Recheck the cross-match.
Give IV steroids.

Question: 5

A general practitioner has recently become aware of several cases of suspected food poisoning among children
from the same school. The next step should be to

notify the local public health authority.


consult with laboratory to verify your impression.
examine the food handlers at the school kitchen.
consult with other general practitioners in your area to verify your impression.
take stool specimens.
Question: 6

A 71-year-old gravida 4, para 4 presents with a 4-year history of worsening urinary incontinence. She leaks
urine just prior to reaching the toilet and whenever she hears running water. Leaking is associated with
urgency. She denies leaking with coughing. Her post-void residual urine is 20 mL. What is the most likely
diagnosis?

Diabetes mellitus.
Stress incontinence.
Overflow incontinence.
Detrusor instability.
Psychogenic incontinence.

Question: 7

A six-month-old previously healthy infant presents with a one-day history of irritability and poor feeding.
Cardiac monitor shows narrow complex tachycardia at 240 beats per minute. Blood pressure is 85 mmHg
systolic and peripheral pulses are palpable. Which of the following is the next most appropriate treatment?

IV epinephrine 0.1ml/kg of 1:10,000.


Defibrillation with 2 joules/kg.
Synchronized cardioversion with 1 joule/kg.
IV bolus of 20 ml/kg normal saline.
IV adenosine 0.1mg/kg rapid push.

Question: 8

An 8-year-old girl presents with symptoms of dysuria, but her urine culture is negative. Which one of the
following diagnoses is most likely?

Enuresis.
Pyelonephritis.
Interstitial nephritis.
Vulvovaginitis.
Hydronephrosis.
Question: 9

A 72-year-old man presents to the Emergency Department complaining of lower abdominal pain of 3 days'
duration. The pain is constant and is located mostly in the lower left quadrant and hypogastric area. He is also
complaining of chills and of a burning sensation when voiding. His stools are normal. Physical examination
shows a temperature of 38.2C. Blood pressure and heart rate are normal. A tender mass in the hypogastric
area, measuring 15 cm x 15 cm, is noted on examination of the abdomen. Rectal examination shows a slightly
enlarged firm prostate. What is the most likely diagnosis?

Sigmoid colon cancer.


Appendicitis.
Diverticulitis.
Urinary retention.
Aortic aneurysm.

Question: 10

Many approaches have been considered to limit increases in the costs of health care. In Canada, governments
have relied most heavily on

introduction of utilization of deterrent fees.


education of the public in the conservative use of health services.
education of health professionals in the conservative use of health resources.
limitations on the supply of hospital funds, physicians, and physician fees.
changes in the methods of paying health professionals.

Question: 11

A psychiatrist planned a randomized clinical trial of a new drug. She expected a success rate of 60%. A 15%
difference in the success rates is the minimum which she regards as having clinical importance. She needs to
enroll about 150 patients for the difference to be significant at a p value < 0.05. Unfortunately, she could only
recruit 100 patients. Although the success rate was close to the expected, the p value was greater than 0.05.
The psychiatrist can conclude that

the trial has failed to demonstrate that the new drug is superior.
the new drug is of no value.
there is no difference between the old and new drugs.
the results justify the use of the new drug routinely.
she has confused clinical and statistical significance in her trial design.
Question: 12

A 44-year-old man presents to your office 3 weeks after the breakdown of his marriage. He complains of low
mood, anorexia, nausea, insomnia, poor concentration, and low self-esteem. Which one of the following
features from the history best suggests that he may benefit from antidepressant therapy?

The patient displays prominent anxiety symptoms.


There was past alcohol abuse.
His level of intelligence is above average.
He is willing to comply with medications.
His mother and sister responded favorably to antidepressant therapy.

Question: 13

A 62-year-old morbidly obese male with type 2 diabetes mellitus is being started on metformin. Which of the
following is the most common drug adverse effect affecting compliance?

Headache
Diarrhea
Weight gain
Edema
Fatigue

Question: 14

An 11-year-old child is brought unconscious to the Emergency Department. You diagnose a ketoacidotic coma
due to undiagnosed diabetes. You want to start insulin treatment but the parents refuse, stating that their beliefs
prohibit the use of non-human derived hormones. Which one of the following is the most appropriate action?

Initiate insulin treatment since you judge it necessary.


Obtain a supporting opinion from a medical colleague to initiate treatment.
Ask the child protection agency to act as a mediator between you and the parents.
Obtain a court order before initiating insulin treatment.
Initiate volume repletion and make sure the child is comfortable.
Question: 15

Five years ago, you completely removed a malignant melanoma on a middle-aged woman. She was disease-
free on follow-up. Today, she presents with her husband who complains that she has been irritable, depressed,
demanding and uncooperative. She looks vague and answers questions slowly. Brief general examination is
within normal limits. Which one of the following management steps is the most appropriate?

Order a computed tomography scan of the head.


Obtain a drug and alcohol screen.
Start a therapeutic trial of antidepressants.
Refer the patient to a counselor.
Order a complete blood count and blood chemistry.

Question: 16

A previously healthy 43-year-old man presents with increasing fatigue, nausea, vomiting and abdominal pain.
On examination, he has a blood pressure of 90/50 mmHg and a heart rate of 112/minute. He has diffuse mild
abdominal pain without masses or rebound. Blood works reveal the following: Sodium (Na) 124 mmol/L
Potassium (K) 5.2 mmol/L Chloride (Cl) 90mmol/L Glucose 4.0mmol/L Creatinine 108 mol/L After the
blood work was drawn, he was given 2 litres of normal saline without improvement in his blood
pressure.Which one of the following will be the next step in your management?

Kayexalate
3rd generation cephalosporin.
Corticosteroids.
Hypertonic (3%) saline.
Fluid restriction.

Question: 17

Your 35-year-old human immunodeficiency virusseropositive patient plans to travel extensively in tropical
developing countries.Which one of the following vaccines should be administered?

Hepatitis B.
Oral cholera.
Yellow fever.
Oral typhoid.
Bacille Calmette-Gurin (BCG).
Question: 18

A 19-year-old primigravida woman presents for her first prenatal visit at 12 weeks' gestation. She is generally
healthy. She has a history of frequent urinary tract infections as a child. Which one of the following is most
appropriate?

Urine culture.
Cystoscopy.
Prophylactic antibiotics.
Renal ultrasound.
24-hour urine for creatinine clearance.

Question: 19

A 16-year-old girl treated with fluoxetine 20 mg per day returns to your office after two weeks complaining
that despite taking her medication every day, she is still having problems with her mood, appetite and
concentration but is not actively suicidal. Upon further inquiry, there is some improvement in her sleep and
energy. She is tolerating the medication. What would be the most appropriate action in addition to continued
monitoring?

Add risperidone, 0.25 mg per day


Increase fluoxetine to 30 mg per day
Add bupropion, 100 mg per day
Continue fluoxetine, 20 mg per day
Switch to venlafaxine, 37.5 mg per day

Question: 20

A 79-year-old woman presents to the Emergency Department with a 1-hour history of tiredness and a heavy
feeling in her chest. She is pale and slightly dyspneic. Auscultation shows a few persistent crackles at the right
pulmonary base, and she has a definite abdominal jugular test. Her pulse rate is 160/minute. The
electrocardiogram shows supraventricular tachycardia. Which one of the following would be the most
appropriate therapy?

Adenosine intravenously.
Digoxin intravenously.
Warfarin and digoxin orally.
Digoxin orally.
Xylocaine intravenously.
Question: 21

A 2-week-old newborn girl with group B streptococcus meningitis is being treated in hospital. On the second
day of therapy, her serum sodium level is 123 mmol/L. Which one of the following is the most likely cause of
this finding?

Drug induced.
Syndrome of inappropriate antidiuretic hormone.
Inadequate sodium intake.
Increased renal sodium loss.
Factitious hyponatremia secondary to hyperglycemia.

Question: 22

A 52-year-old man presents with tearing chest pain radiating to his back. His past medical history is significant
for hypertension. Physical examination reveals a blood pressure of 190/80 mmHg in the right arm and 220/110
mmHg in the left arm and heart rate of 100 beats/minute. An electrocardiogram is normal. Which one of the
following would be the most appropriate first step?

IV heparin
Consult a cardiovascular surgeon
Order creatine kinase and troponin levels
IV labetalol
Echocardiogram

Question: 23

You are a urologist and frequently receive samples of sildenafil (Viagra) which are sometimes left in the
examination room by industry representatives. You are told by a reliable source that one of your patients is
selling ""Viagra"" at rave parties. Which one of the following should you do?

Refuse to see the patient again.


Call the patient's pharmacy and advise them to stop dispensing medications to the patient.
Send an e-mail to all urologists in your town to warn them about the patient.
Call the police and advise them of the situation.
Discuss your suspicions with your patient.
Question: 24

A 10-year-old girl presents with neck swelling. She has had no symptoms of hyperthyroidism. Examination
reveals a lobular goiter. A diagnosis of Hashimoto thyroiditis is suspected. Which one of the following
investigations is most appropriate to confirm this diagnosis?

Thyroid antiperoxidase antibodies (TPO Abs).


Thyroid biopsy.
Thyroid-stimulating hormone level.
Thyroxine (T4) level.
Triiodothyronine (T3) level.

Section: 3

Question: 1

A 55-year-old male patient with suspect tuberculosis requires diagnostic bronchoscopy. Which one of the
following statements about informed consent is most appropriate?

No consent is required since diagnostic studies for suspect tuberculosis may be ordered under public
health legislation if necessary.
An experienced nurse may explain the procedure and sign the consent form.
The consent must include information regarding all side-effects of the procedure.
The consent form must be signed by a witness other than the physician.
The interview with the patient must be conducted by the physician who will carry out the procedure
or by another equally competent physician.

Question: 2

A middle-aged man presents to your office with recurrent, generalized, dull headaches associated with nausea
and vomiting. On examination, he looks depressed and has poor concentration and memory problems.
The physical is normal. Which of the following is the most likely diagnosis?

Pseudodementia.
Somatoform pain disorder.
Intracranial space occupying lesion.
Major depression.
Chronic alcoholism.
Question: 3

Which one of the following findings on pelvic examination is compatible with a diagnosis of procidentia?

The uterus descends below the introitus with or without the patient standing.
An ulcerated solid mass is visible at the introitus.
The cervix is elongated to such an extent that it readily protrudes beyond the introitus when the
patient is asked to "bear down".
A visible lump is seen to protrude through the introitus when the patient is asked to "bear down".
The patient is asked to "bear down" during pelvic ultrasound to confirm that it is the uterus that
descends and not some other structure.

Question: 4

A 58-year-old postmenopausal woman complains of backache. An x-ray of the lower spine reveals a fracture
in a lumbar vertebra. Which one of the following tests would best quantify her risk for osteoporosis?

X-ray of wrist.
Serum estradiol.
Serum calcium.
Bone densitometry.
Urinary calcium.

Question: 5

Pruritic, flat-topped, violaceous papules and plaques around wrists, ankles, and mucous membranes in a 52-
year-old alcoholic are most compatible with the diagnosis of

lichen planus.
vitiligo.
rosacea.
psoriasis.
scabies.
Question: 6

Patients should be instructed to avoid foods with a high tyramine content when taking which one of the
following drugs?

Moclobemide.
Fluoxetine.
Desipramine.
Nortriptyline.
Phenelzine.

Question: 7

A retrospective cohort study is conducted to evaluate the relationship between cigarette smoking and bladder
cancer in men. In the study, 48 out of 10,000 smokers and 24 out of 20,000 non-smokers develop bladder
cancer. The risk difference is:

24.
0.25.
4.
0.0036.
36.

Question: 8

Which one of the following is a physician obliged to do if and when requested by a coroner or medical
examiner?

Review the records of a colleague who is under investigation in relation to a suspicious death.
Investigate the medical cause of a death.
Supervise the practice of a colleague who has recently undergone rehabilitation for alcoholism.
Act as a coroner temporarily.
Provide the original office chart of a patient.
Question: 9

A 65-year-old woman, who is otherwise well, has disabling rheumatoid arthritis. You elect to treat her with the
nonsteroidal anti-inflammatory agent (NSAID) ibuprofen in an attempt to reduce the inflammatory response
and pain she suffers. Which one of the following side effects of NSAIDs is most likely to develop in this
patient?

Gastritis.
Thrombocytopenia.
Duodenal ulcer.
Acute renal failure.
Congestive heart failure.

Question: 10

A 21-year-old patient complains of clear vaginal discharge, which usually occurs midcycle. On examination
the vaginal pH is 4. The wet mount and "whiff test" are negative. Which of the following would you expect to
find on examination?

The sqamocolumnar junction 1cm outside the external os (ectropion).


A "cottage cheese" like discharge in the vagina.
A fishy odor.
Petechiae on the vaginal mucosa and cervix (strawberry cervix).
Vaginal condyloma.

Question: 11

A surgeon directs some medical trainees to perform a bimanual pelvic examination on a patient who is under a
general anesthetic. Most of the group complies with the surgeon's instructions. A trainee who knows this was
performed without informed consent should

ignore the situation as no harm was done to the patient.


discuss the incident with the patient.
seek protection under provincial/territorial whistle-blower laws.
report the surgeon's conduct to an appropriate superior.
attempt to change the surgeon's behavior.
Question: 12

A 21-year-old man is involved in a motorcycle collision. On arrival in the Emergency Department, he is noted
to have no movement of his legs, no sensation in his legs and loss of rectal tone. Which one of the following
may be beneficial?

Revascularization of the spinal cord


High dose steroids
High dose antihistamines
Spinal cord cooling
Opioid antagonists

Question: 13

An 85-year-old gravida 5, para 5 presents to your office with difficulty voiding and dysuria for 6 months'
duration. On examination, you observe the cervix at the level of the introitus. Which one of the following
would be the best initial management?

Oral antibiotics
Vaginal hysterectomy
Pelvic floor exercises
Insertion of a pessary
Oral anticholinergics

Question: 14

A 6-year-old child is brought to your office because of a painful red eye. On examination, the child has a
cloudy anterior chamber and circumlimbal redness. Which one of the following is the most likely diagnosis?

Acute uveitis.
Foreign body.
Corneal abrasion.
Acute conjunctivitis.
Orbital cellulitis.
Question: 15

A 75-year-old man is brought to your office for assessment of increasing forgetfulness and confusion. Which
one of the following features is most suggestive of a diagnosis of normal pressure hydrocephalus?

Myoclonic movements.
Broad-based gait with hesitant initiation of walking.
Family history of similar problems.
Enlargement of the fourth ventricle.
Diffuse slowing in the electroencephalogram.

Question: 16

Compared with control subjects, patients with major depression generally show

shortened REM time and no change in stage 4 sleep.


frequent REM periods and decreased density.
decreased REM latency and decreased stage 4 sleep.
increased rapid eye movement (REM) latency and increased stage 2 sleep.
shortened REM time and decreased stage 2 sleep.
Question: 17

A 7-year-old boy with a past history of chronic kidney disease due to posterior urethral valves diagnosed at
birth presents to the Emergency Department with a 48-hour history of diarrhea and vomiting. He weighs 30 kg
and his height is 130 cm. Examination reveals a pale child with cool hands and feet and a prolonged capillary
refill time. Laboratory results showed:

Sodium 134 mmol/L


Potassium 5.2 mmol/L
Bicarbonate 19 mmol/L
Creatinine 170 mol/L
Urea 17.5 mmol/L

Which one of the following would be the most appropriate to order as an intravenous bolus to treat this acute
illness?

500 mL of 0.45% saline.


600 mL of 5% dextrose 0.45% saline.
400 mL of 0.9% saline.
300 mL of 0.2% saline with 20 mmol/L KCl.
150 mL of 0.2% saline with 10 mmol/L KCl.

Question: 18

A gravida 2, para 1 woman presents to your office for her first prenatal visit. She is currently 8 weeks'
pregnant. She had a previous cesarean section at term for breech presentation. Her first pregnancy was
otherwise uneventful. She would like to discuss her delivery options. Which one of the following would you
tell her?

Uterine rupture only occur during labor.


The first sign of uterine rupture is scar pain.
Induction of labor with prostaglandins is associated with an increased risk of uterine rupture.
Oxytocin augmentation is not safe in women undergoing a trial of labor.
Post dates is a contraindication to a trial of labor.
Question: 19

A 26-year-old nurse with a previously negative tuberculin skin test is found to have a positive reaction to
purified protein derivative (PPD) 3 months after having been exposed, in hospital, to a patient with active
tuberculosis. The nurse is asymptomatic and a chest x-ray shows no abnormality. Which one of the following
would be the correct management at this time?

Isoniazid and ethambutol until sputum or gastric wash cultures are reported negative in 8 weeks.
Isoniazid and ethambutol for 12 months.
Sputum or gastric washings for tuberculosis cultures and initiate therapy only if these prove positive.
Isoniazid for 6 months.
X-rays of the chest every 3 months and initiate treatment should a pulmonary infiltrate develop.

Question: 20

A 62-year-old asymptomatic man who received radiation treatment for acne during adolescence is seen for
evaluation of his thyroid gland. On examination, the thyroid gland is non-tender and normal in size. A 2 cm
nodule is palpable. There is no lymphadenopathy. Which one of the following courses of action is most
appropriate?

Hemithyroidectomy.
Fine-needle aspiration biopsy.
Radioiodine thyroid scan.
Determination of serum thyroid-stimulating hormone.
Ultrasound and reassessment in 6 months.

Question: 21

Which one of the following predicts a poor outcome for schizophrenia?

Insidious onset.
Being married.
Good premorbid social functioning.
Family history of mood disorders.
Presence of a precipitating factor.
Question: 22

You have discovered, by chance, conclusive evidence that one of the physicians in your partnership is
submitting bills to the provincial/territorial medical services office for services that he did not perform. Which
one of the following should you do?

Notify the provincial/territorial licensing authority.


Call a partnership meeting to discuss your concerns.
Notify the local medical association.
Confront your colleague with this information.
Notify the police.

Question: 23

A 74-year-old man complains of muscle weakness, especially in the thigh muscles, and stiffness in his
shoulders. Which one of the following findings is most suggestive of a diagnosis of polymyalgia rheumatica?

Weakness of the levator palpebrae.


Elevated erythrocyte sedimentation rate.
Muscle tenderness.
Elevated serum creatine kinase.
Evidence of muscle damage on electromyogram.

Question: 24

A 17-year-old female with a recent history of suicide attempt (wrist slashing) walks in to your office
complaining of headache and insomnia. Her boyfriend recently left her and she is failing at school. She admits
to drinking more on the weekends with her friends. Her father is currently unemployed and is drinking
""again"". Which one of the following is the most appropriate in the initial management of this patient?

Express concern about the patient's risk of suicide and ask specifically about her intent.
Refer the patient to the Emergency Department for psychiatric evaluation.
Discuss how her father's drinking affects her and suggest a referral to alcohol services.
Discuss her school problems and offer to call the guidance counsellor.
Reassure the patient that her call for help is heard and arrange for regular counselling.
Section: 4

Question: 1

Which one of the following features differentiates posttraumatic stress disorder from adjustment disorder?

Co-morbid anxiety with irritability.


The presence of autonomic symptoms.
The presence of sadness.
The absence of concomitant disease.
The severity of the precipitating stressor.

Question: 2

Four weeks ago, a 42-year-old woman presented to your office having already started taking St. John's Wort
300 mg 3 times a day as recommended by her naturopath. She had a two-month history of feeling sad and
distant from her family, crying spells when alone and difficulty falling asleep. She continued to work part
time. Although she continued to take St. John's Wort 300 mg TID, she now reports that she feels more
distracted, has sleepless nights and more frequent crying spells. She is not suicidal. In addition to regular
follow up, which one of the following is the most appropriate management plan?

Add Omega-3 5 g/day to the current dose of St. John's Wort.


Stop St. John's Wort and begin venlafaxine (Effexor XR) 75 mg QAM, after a wash out period.
Continue St. John's Wort at the current dose and add paroxetine (Paxil) 20 mg OD.
Add Lithium 600 mg daily to augment her current dose of St. John's Wort.
Increase the dose of St. John's Wort to 600 mg TID.

Question: 3

A worried mom brings in her nine-year-old son for assessment due to repeated facial, neck and shoulder
movements. Which characteristic is most useful in supporting a diagnosis of benign tic disorder of childhood?

No past history of meningitis or encephalitis.


Absence of vocal tics.
Tic-free episodes of less than three consecutive months.
Age of onset.
Absence of a learning disorder.
Question: 4

A 28-year-old housewife presents with a 12-week history of intermittent episodes of slightly raised, itchy,
pink, skin lesions on her trunk and limbs. She is not aware of any associated symptoms or precipitating factors.
The process resolves completely and spontaneously over 24 hours. Which one of the following is the most
appropriate statement about testing for the etiology of this process?

An antinuclear antibody is often positive.


Skin tests will identify contact allergens.
An elimination diet will identify offending foods.
Blood tests usually fail to identify specific causes.
A skin biopsy will identify urticarial vasculitis.

Question: 5

A 6-year-old girl is going to summer camp. Her parents come to your office seeking advice as she had a severe
reaction to a honey bee sting 2 years ago. Which one of the following recommendations should be given?

The child should wear insect repellent.


Self-administered epinephrine should be carried by the child at all times.
The child should eat indoors at all times.
If stung, the child should be given an antihistamine then be observed.
A tourniquet above the site of the sting is of little value.

Question: 6

A 60-year-old woman complains of progressive hearing loss for the last 5 years without any previous trauma.
She tells you that hearing loss tends to run in the family. You suspect otosclerosis. Which one of the following
findings is most in favor of the diagnosis of otosclerosis?

Perforated eardrum.
Destruction of external auditory canal.
Air fluid level behind the eardrum.
Normal eardrum.
Clear otorrhea.
Question: 7

You are about to see an 18-month-old boy for a well child check-up. The nurse expresses concern to you that
the baby looks a bit thin. The most accurate method of assessing failure to thrive in this child would be to:

weigh him weekly to check for weight loss.


check whether his weight is below the 5th percentile for his age.
screen for developmental delay.
compare his weight to that of his older brother at age 18 months.
check the graph of his weight since birth.

Question: 8

A 55-year-old man is being started on spironolactone for congestive heart failure. Which one of the following
side effect of this medication should he be counseled about?

Hair loss
Hypocalcemia
Acne
Gynecomastia
Hypercalcemia

Question: 9

A woman, gravida 2, para 1, consults at 10 weeks of gestation. In obtaining her history, which of the following
is the most important risk factor for preterm delivery?

She works as a sales person in a department store.


She has a history of anorexia nervosa.
Her first baby was born at 34 weeks.
She is 36 years old.
She was treated for Chlamydia 6 months ago.
Question: 10

A 67-year-old man presents with a painless right groin mass. He first noticed it a month ago and it seems to
have grown slightly since. History includes a coronary balloon angioplasty 3 months ago. On physical
examination, the mass is hard, fixed, and slightly pulsatile. Which one of the following would confirm the
diagnosis?

Needle biopsy
Angiography
Computed tomography scan
Ultrasound
Open biopsy

Question: 11

Which one of the following is the most common cause of legal action being brought against physicians and
hospitals?

Incorrect diagnosis.
Negligence.
Delayed diagnosis.
Incompetence.
Breakdown in communication.

Question: 12

A 35-year-old woman presents with severe, upper abdominal pain associated with nausea and vomiting.
Examination shows guarding and rebound of both right and left upper quadrants. Her pulse is 120/minute, and
her blood pressure is 90/60 mm hg. Her blood work includes a white blood cell count of 25x109/L and an
amylase of 2000 U/L. Which one of the following is the most likely cause of this woman's shock?

Disseminated intravascular coagulation


Loss of whole blood into the peritoneal cavity
Sepsis resulting from Gram-negative bacterial growth in necrotic pancreatic tissue
Loss of fluid into the retroperitoneal space
Deleterious effects of trypsin and other toxic products on cardiac function
Question: 13

A 32-year-old woman presents to the emergency complaining of feeling anxious, short of breath, nauseated
and dizzy. She is sweaty and complains of feeling "pins and needles" in her hands. Upon questioning, she
describes the room as spinning. She is certain that something terrible is happening to her "like a heart attack."
Which one of the following symptoms should alert you to a medical condition presenting as a panic attack?

Dyspnea
Fear of dying
Paresthesias
Vertigo
Diaphoresis

Question: 14

Manual compression of a small, painful mass beneath the nipple in a 30-year-old woman causes a bloody
nipple discharge. The most likely diagnosis is

abscess.
fibroadenoma.
fibrocystic disease.
Paget disease of the nipple.
intraductal papilloma.

Question: 15

A 5-year-old boy presents with periorbital edema. Which one of the following is the best initial management?

Urinalysis (urine microscopy).


Look for other signs suggestive of allergy.
Referral.
Trial of oral antihistamine/decongestant.
Sinus x-rays.
Question: 16

A 24-year-old woman has developed the papillary lesions shown in the referenced illustration. Which one of
the following is the causative organism?

Human papillomavirus.
Treponema pallidum.
Molluscum contagiosum.
Chlamydia trachomatis.
Pthirus pubis.

Question: 17

A 30-year-old man presents with irritability and insomnia. He impulsively quit his job after arguing with his
boss. Although he has limited experience, he is confident that he will succeed if he starts his own company. He
has a history of major depression and alcohol abuse. During the consultation, he looks agitated and expresses
suicidal ideation. Which one of the following is the most likely diagnosis?

Substance-induced mood disorder.


Bipolar disorder, mixed episode.
Brief psychotic disorder.
Adjustment disorder with disturbance of emotions and conduct.
Antisocial personality disorder.

Question: 18

A patient with suspected overdose presents with the following physical signs: pulse 72/minute, blood pressure
110/70 mm Hg, respiratory rate 12/minute, somnolence and myotic pupils. The skin and mucous membranes
are normal as well as the deep tendon reflexes. Which one of the following drug classes would be the most
likely cause of this overdose?

Cholinergics.
Tricyclics.
Opiates.
Salicylates.
Sedative-hypnotics.
Question: 19

A 20-month-old infant presents with a fever and refusal to eat solid food. Lesions of the type seen in the
referenced illustration are also present in the interdigital spaces of the fingers. Which one of the following is
the most likely diagnosis?

Coxsackievirus A infection.
Sarcoptes scabiei infestation.
Varicella.
Beta-hemolytic streptococcal infection.
Rickettsia rickettsii infection.

Question: 20

A 10-month-old native Canadian (Aboriginal) is flown to your hospital because of a fever of 38.5C and a stiff
neck. Which one of the following investigations is the best way to differentiate an aseptic meningitis from a
tuberculous meningitis when examining the cerebrospinal fluid?

Protein content.
Presence of mononuclear leucocytes.
Latex agglutination.
Total cell count.
Gram stain.

Question: 21

The presence of pyometra in a postmenopausal patient is suspicious for

chronic cervicitis.
endometrial carcinoma.
atrophic endometritis.
an endocervical polyp.
pelvic tuberculosis.
Question: 22

A 70-year-old woman presents to your office with a new onset of postcoital vaginal bleeding. She has not been
on hormone replacement therapy since her menopause 19 years ago. Which one of the following is the most
likely diagnosis?

Cervical intraepithelial neoplasia.


Ovarian cancer.
Endometrial hyperplasia.
Endometrial cancer.
Atrophic vaginitis.

Question: 23

A 32-year-old man is seen in your office with a 2-month history of rectal pain and rectal bleeding. Which one
of the following is the most llikely diagnosis?

Rectal cancer
Ischiorectal abscess
Anal fissure
Internal hemorrhoids
Intersphincteric abscess

Question: 24

A 14-year-old female presents to your office with vague complaints of headaches and stomach aches. She
informs you that for three weeks she has been feeling moody, bored, tired and wants to sleep all day. She
broke up with her boyfriend four weeks ago and she is fighting a lot with her parents. She has trouble
concentrating at school and uncharacteristically failed her last math test. Which one of the following would be
the most appropriate intervention?

Recommend cognitive behavior therapy.


Recommend a psychometric assessment.
Initiate family therapy.
Refer to a gastroenterologist.
Prescribe a benzodiazepine.
Self-Administered QEI-MCQ Exam 3
Section: 1

Question: 1

A 3-year-old girl is brought by her parents with concerns about her left eye. On examination, there is no tearing,
no conjunctival erythema nor discharge. Pupillary reactions are normal and extraocular movements are full. The
red reflex is present in both eyes. On cover/uncover testing, there is marked abnormal movement of the left eye.
Which one of the following is the most likely diagnosis?

Corneal abrasion.
Strabismus.
Cataract.
Amblyopia.
Retinoblastoma.

Question: 2

Which one of the following must be proved for a malpractice action against a physician to succeed?

The patient did not contribute to a poor outcome.


The patient did not receive the "best possible care".
The action has been completed within the period allowed by the statute of limitations.
The physician made an error in judgement.
The patient suffered harm or injury.

Question: 3

A 24-year-old gravida 7, para 6 woman presents to a tertiary care hospital in active labor at 37 weeks' gestation.
Pelvic examination shows bulging membranes with a cervical dilatation of 7 cm. No presenting part is felt in
the pelvis. Which one of the following would be the best immediate management?

Immediate emergency lower segment cesarean section.


Artificial rupture of the membranes in the operating room.
Administration of oxytocin to engage a presenting part.
Intermittent fetal monitoring and immediate pelvic examination if the membranes rupture.
Real-time ultrasound on the labor floor to determine the presenting part of the fetus.

Question: 4

Which one of the following cardiovascular side effects may be attributed to tricyclic antidepressants at
therapeutic doses?

Orthostatic hypotension.
Bradycardia.
QRS narrowing.
Acceleration of intraventricular conduction.
Ectopic beats.
Question: 5

A 12-year-old girl presents with a 2-day history of jaundice. She has had mild fatigue for a month. On
examination, she has icterus and mild, non-tender hepatomegaly. Laboratory investigations reveal the
following:

Total bilirubin 87 mol/L


Conjugated bilirubin 72 mol/L
Alanine aminotransferase (ALT) 276 U/L
Aspartate aminotransferase (AST) 215 U/L
Alkaline phosphatase 97 U/L

Serum immunoglobulin G (IgG) level is markedly elevated. High titers of anti-nuclear antibody and anti-
smooth muscle antibody are present. A complete blood count and serum albumin are normal. Which one of the
following is the most likely diagnosis?

Autoimmune hepatitis
Primary biliary cirrhosis
Systemic lupus erythematosus
Choledocholithiasis
Wilson's disease

Question: 6

A 60-year-old man presents with progressive weakness of his arms and legs. He reports difficulty climbing
stairs or combing his hair. He also has difficulty swallowing, but he has no visual complaints. On physical
examination, you note a maculopapular eruption on the eyelids, nose, cheeks and knuckles. Joint examination is
normal. Which one of the following is the most likely diagnosis?

Rheumatoid arthritis.
Dermatomyositis.
Systemic lupus erythematosus.
Myasthenia gravis.
Polymyalgia rheumatica.

Question: 7

Which one of the following treatments is most likely to exacerbate a depressive disorder in an 82-year-old
woman with rheumatoid arthritis?

Penicillamine.
Acetylsalicylic acid.
Gold salts.
Naproxen.
Prednisone.
Question: 8

A 65-year-old woman has a significant history of heart disease. She has been taking continuous combined
hormone replacement therapy and has read in the press that there are increased risks with hormones. Which one
of the following would you recommend?

Stop the progestins and continue the estrogens at a higher dose.


Continue the hormones to prevent heart disease.
Change her replacement to transdermal patches of combined hormones.
Stop the estrogens and continue the progestins.
Wean off the hormones gradually as they are only indicated for significant symptoms of menopause.

Question: 9

Which one of the following occupational groups is not subject to mandatory reporting of unfitness for duty by
their physicians?

Police officers.
Airline pilots.
Air traffic controllers.
Railway engineers.
Transport truck drivers.

Question: 10

Acute polyhydramnios associated with fetal esophageal atresia at 37 weeks' gestation is best managed by
cesarean section.

administration of diuretics to the mother.


administration of indomethacin to the mother.
amniocentesis and drainage of excess fluid.
strict bedrest and fluid restriction.

Question: 11

A young child with fever and convulsion which lasted 7 minutes has just been admitted to the Emergency
Department. Which one of the following features would be against the diagnosis of a simple febrile convulsion?

The seizure was stopped with lorazepam.


She had a similar episode 4 months ago.
She was incontinent of urine.
She is 2 years old.
The seizure involved the right arm and right leg.
Question: 12

Disclosures made by patients to their doctors are

sacrosanct.
confidential.
contractual.
privileged.
fiduciary.

Question: 13

A 25-year-old man presents as odd, maintaining no eye contact. He is having difficulties obtaining a job, as he
gets preoccupied with his interest in royalty. He does not report any magical thinking. He has a longstanding
history of resistance to change, hypersensitivity to noises and minimal social contact. Which one of the
following is the most likely diagnosis?

Asperger's disorder
Obsessive compulsive disorder
Social phobia
Schizophrenia
Generalized anxiety disorder

Question: 14

Behavior modification works best when

the desired goal of changed behavior is clearly defined.


insight needs to be developed to change behavior.
family and other caretakers are unable to cooperate.
the environment is an unpredictable trigger or stimulus to behavior.
behavioral dysfunction is longstanding.

Question: 15

Most low-risk pregnancies at term will be monitored with intermittent auscultation of fetal heart rate during
labor. Which of the following is an indication for performing continuous electronic fetal monitoring?

Suspected macrosomia.
Use of prostaglandins for induction of labor.
Primiparity.
Recurrent decelerations heard after contractions.
Maternal use of morphine for prolonged latent stage.
Question: 16

You observe a serosanguinous discharge from the wound of a patient who underwent a sigmoid resection five
days earlier. Which one of the following is the most likely diagnosis?

Anastomotic leak.
Fascial dehiscence.
Necrotizing fasciitis.
Wound infection.
Ureteric injury.

Question: 17

You are working as a student in an orthopedic surgeons office and overhear a conversation between the surgeon
and another medical colleague. The caller is requesting that their parent be prioritized to the top of the surgical
list to get a hip replacement earlier than everybody else. The orthopedic surgeon agrees to do so as a personal
favour, despite overtly admitting that this patient is not as urgent as most of his other waiting patients. Which
one of the following is the most appropriate management?

Report this to the wait list manager.


Express your disapproval and leave the office.
Discuss the issue with the surgeon.
Say nothing at this time.
Report this incident to the chief of staff.

Question: 18

During a 1-year period, you diagnose three new cases of malignant melanoma among patients in your practice.
Which one of the following is the most appropriate confirmation of a cluster?

The observed number of cases in your practice during the past year is significantly higher than
during previous years.
All of the cases have lived in the same neighborhood for several years.
They are all above age 50 and have worked outdoors.
They are all white men.
The observed number of cases in the area is significantly higher than the expected number in a
reference population.

Question: 19

A 78-year-old non-smoking woman presents to her general practitioner with a history of nocturnal cough for
almost 1 year, and a recent history of fever, sweats and production of foul-smelling sputum. Aside from
antacids for heartburn, she does not take any medication. On examination, she is ill, febrile, edentulous, and has
signs of consolidation in the apical segment of the right lower lobe. Which one of the following is the most
probable etiology of her condition?

Histoplasmosis.
Esophageal reflux.
Squamous cell carcinoma.
Pulmonary tuberculosis.
Sarcoidosis.
Question: 20

A 14-year-old boy is seen in the Emergency Department with painless gross hematuria and left-sided abdominal
pain. His uncle died of a brain hemorrhage at age 58. Which one of the following is the most likely diagnosis?

Acute glomerulonephritis.
Urinary tract infection.
Polycystic kidney disease.
Nephrolithiasis.
Renal contusion.

Question: 21

A 16-year-old girl presents with amenorrhea. On examination, you notice lanugo hair. Her body mass index is
15. Which one of the following is the most likely diagnosis?

Bulimia.
Thyroid disorder.
Pituitary adenoma.
Anorexia nervosa.
Pheochromocytoma.

Question: 22

A man has made a clinic appointment to discuss his father's condition. He works abroad and has not seen his
father for 3 years. Since his arrival, he has noted that his father shows signs of memory loss which he finds
alarming. He has a banking power of attorney and has observed that some of his father's banking records are not
updated. The patient lives alone at home. He has been seen by the same physician for years. This doctor has
noted the patient's cognitive impairment over the past 2 years with investigation suggesting the possibility of
Alzheimer disease. The patient has asked his physician not to disclose anything to his son, because he is afraid
of being placed in a home. What should the physician do?

The physician must not disclose the information because this would be prejudicial to the father-son
relationship.
The physician must disclose the information to the son because the latter has a power of attorney.
The physician must disclose the information because the facts reported by the son demonstrate that
the patient is impaired.
The physician must not disclose the information because it would undermine trust in the doctor-
patient relationship.
The physician must disclose the information, otherwise he would be contravening the beneficence
principle.
Question: 23

Two months ago, you were asked to complete government tax forms for the family of a 12-year-old child with
cystic fibrosis so that they will receive financial aid. You have not yet filled out these forms. The mother of the
child phones to enquire about the application. Which one of the following is your best course of action?

Explain how busy you are to the mother.


Quickly complete the forms and tell the mother they have been sent.
Ask the mother to resend the forms, as they have been misplaced.
Apologize for the delay and complete the forms today.
Have your receptionist fill out the forms for you to sign.

Question: 24

Which of the following warnings characterizes transient ischemic attacks in vertebrobasilar insufficiency?

Double vision, slurred speech, dizziness, ataxia and numbness around the lip or face.
Transient aphasia.
Reports of a "shade coming down" over the eyes.
Transient ipsilateral blindness.
Motor and sensory symptoms in a single extremity (upper or lower) or a clumsy "bear's paw" hand.

Section 2

Question: 1

A hyperdynamic, hypermetabolic state, along with low vascular resistance, is an early feature of which one of
the following?

Acute adrenocortical insufficiency.


Massive pulmonary embolism.
High spinal anesthesia.
Hypovolemic shock treated with alpha adrenergic drugs.
Septic shock.

Question: 2

A 50-year-old man presents at your office regarding management of his hypertension. You take a history,
perform a physical examination, and review his medical records. Which one of the following justifies a further
work up seeking a secondary cause of his hypertension?

Palpitations
Cerebrovascular accidents in both parents
A carotid bruit
An S4 on cardiac auscultation
Hypokalemia
Question: 3

You examine a 5-year-old boy with a 3-week history of limping with pain in the right knee. You find that he has
limitation of hip movement. An x-ray of the right hip reveals flattening of the femoral head with bone
remodeling and widening of the hip joint space. What is the most likely diagnosis of this condition?

Avascular necrosis of the femoral head.


Osteomyelitis of the hip joint.
Juvenile rheumatoid arthritis.
Slipped femoral epiphysis.
Transient synovitis of the hip.

Question: 4

A 33-year-old woman presents to her family doctor complaining of chronic pelvic and abdominal pain as well
as sleep and appetite disturbances. She perceives her health is generally poor and she has felt unwell all her life.
Previous investigations have all been negative. She looks generally well and thorough physical examination is
negative. Which one of the following should be done next?

Refer for psychotherapy for depression.


Repeat radiological investigation of gastrointestinal tract.
Prescribe antidepressant medication.
Discuss ongoing or past domestic violence.
Conduct further gynecological assessment.

Question: 5

A competent 63-year-old woman with amyotrophic lateral sclerosis (Lou Gehrig disease) has written an
advance directive stating that tube feeding is not acceptable. She has now suffered a stroke and cannot
communicate or swallow. Her neurological condition is stable, and she is referred to the nutritional support
team. Which one of the following should they do?

Suggest that she have small regular amounts of liquids by mouth.


Insert a nasogastric tube for nutrition.
Recommend that she have compassionate terminal care.
Insert a central line for parenteral alimentation.
Suggest a referral to a surgeon for consideration of a gastrostomy.

Question: 6

A 7-year-old child presents with a 1-day history of high fever and a severe sore throat. Which one of the
following physical findings most likely suggests streptococcus A pharyngitis?

Anterior cervical lymphadenopathy.


Postauricular lymph nodes.
Conjunctivitis with photophobia.
Red vesicular rash.
Koplik spots.
Question: 7

A 7-year old girl sustained a displaced left humeral supracondylar fracture. She is unable to actively extend the
thumb and the metacarpophalangeal joints of her fingers. Sensation is decreased in the first web space. Which
one of the following has she injured?

Anterior interosseous nerve.


Median nerve.
Metacarpophalangeal joints.
Extensor tendons of the fingers and thumb.
Radial nerve.

Question: 8

A 72-year-old woman living in a nursing home and suffering from dementia is treated for a psychotic condition.
Her physician prescribes haloperidol 0.5 mg twice daily. While seeing her at the nursing home, the physician
notices extrapyramidal signs including unsteadiness with a slow gait. The chart indicates that she has been
receiving 5 mg of haloperidol twice daily. The physician stops the medication. The next day, the patient falls
and breaks her hip. Worried, the daughter asks the physician why her mother fell. Which one of the following
statements is the most appropriate?

The physician does not have an obligation to disclose the mistake because it was corrected before the
incident occurred.
The physician must tell the daughter what happened.
The physician cannot disclose a mistake which is not his own.
The physician explains that her mother fell because the drug frequently causes extrapyramidal side
effects.
The physician must disclose the mistake only if the patient's daughter asks about it.

Question: 9

A 76-year-old patient has been complaining of pain in the mouth of 3 months' duration. The pain is worse when
he uses his dentures, and it is located on the lower right gingiva, near the molar region. Examination reveals a
flat, friable ulcer on the alveolar ridge. The dentures recently have become ill-fitting. Which one of the
following diagnoses is the most likely?

Vincent angina.
Traumatic ulcer.
Squamous cell carcinoma.
Dental abscess.
Adenocarcinoma.
Question: 10

A 46-year-old woman presents to the Emergency Department with a 2-day history of heavy vaginal bleeding.
Pelvic examination reveals a bulky uterus and bleeding through the cervical os. Which one of the following
investigations is the most appropriate at this time?

Computed tomography scan of the pelvis.


Pelvic ultrasound.
Endometrial biopsy.
Urine pregnancy test.
Coagulation profile.

Question: 11

A mother brings her 4-week-old baby to your office for examination. She complains that he does not drink
much and that she does not know what to do. His birth weight was 3.5 kg. His present weight is 3.6 kg. Which
one of the following is the most appropriate course of action?

Order a complete blood count and a sweat chloride test.


Change the milk formula for an iron-enriched formula.
Evaluate caloric intake and parent interaction.
Reassure the mother that some children do not gain much weight during the first month of life.
Refer mother and child to a social worker for evaluation and give another appointment in 1 week.

Question: 12

A 24-year-old woman has had watery rhinorrhea, sneezing, tearing, nasal congestion and dysosmia occurring
periodically for the past few years. Which one of the following additional findings is most characteristic of
allergic rhinitis?

Apical tooth abscess.


Nasal polyps.
High arch palate.
Deviated nasal septum.
Frontal sinus atresia.

Question: 13

Which one of the following statements regarding benzodiazepine hypnotics is most accurate?

Rebound insomnia is rarely a problem with shorter half-life benzodiazepines.


Benzodiazepines currently available are similar in terms of efficacy and safety.
They are safe if taken daily as prescribed over a period of 6 months.
Their method of action is poorly understood.
Short-acting benzodiazepines preserve the normal sleep pattern.
Question: 14

A 62-year-old hypertensive man treated with a triamterene/hydrochlorothiazide combination has recently


returned from vacation. He presents with a 2-day history of severe pain in the right ankle, the site of a remote
injury. On examination, the skin over the joint is shiny, tight and dusky red, and the joint is very swollen.
Palpation and both active and passive movements cause exquisite pain. He has a temperature of 38.6C, but the
rest of the physical examination is normal. Which one of the following diagnostic tests is the most appropriate?

Human leukocyte antigen B27.


Urethral swab.
Serum uric acid.
Stool culture.
Joint aspiration.

Question: 15

A 2 kg male infant, born after 37 weeks' gestation, appeared well until his first feeding 2 hours post-delivery.
He regurgitated the feeding and became cyanotic. Nasotracheal aspiration affected improvement. Which one of
the following diagnoses is most likely?

Congenital diaphragmatic hernia.


Congenital vascular ring.
Pyloric stenosis.
Esophageal atresia.
Annular pancreas.

Question: 16

A 29-year-old man comes to the clinic complaining of fatigue. On taking a personal history, you note that he
lives alone and is employed at night as a laboratory technician. He is reclusive and does not socialize. He has
never had a significant relationship. He seems detached, aloof and indifferent to his isolation. His mental status
is otherwise unremarkable. Which one of the following is the most likely diagnosis?

Avoidant personality disorder.


Social phobia.
Schizotypal personality disorder.
Asperger disorder.
Schizoid personality disorder.

Question: 17

Functional inquiry, on an athletic 25-year-old nulliparous woman admits to occasional urine staining of her
underwear after jogging over a hilly terrain. Which one of the following is the most appropriate management?

Instruction in the use of a set of increasing weighted vaginal cones.


Instruction in high intensity pelvic floor exercises 3 times a day for 6 weeks.
Reassurance that this is not unusual and that no treatment is necessary.
Prescription of vaginal estrogen cream to be used daily for the next 3 weeks.
Referral for a course of electrical stimulation treatments to the pelvic floor.
Question: 18

You are the family physician for a 52-year-old man with known coronary artery disease. He recently underwent
cardiac catheterization and stenting of his left anterior descending artery. He comes to see you 2 weeks after the
procedure and confesses that he has not been taking any of the prescribed medications because he is unable to
afford them, as he works at a minimum wage job. His daughter has accompanied him today and informs you
that she has full drug coverage and asks that you re-write the prescriptions in her name. Which one of the
following is the most appropriate response to this request?

Agree to write the prescriptions in the daughter's name and suggest to the patient that he apply for a
provincial drug card.
Decline to write the prescription in the daughter's name.
Agree to write the prescription and document a visit on the daughter's medical chart.
Decline to write the prescription in the daughter's name, except for the more expensive medications.
Agree to write the prescriptions in the daughter's name, but do not provide any refills.

Question: 19

A 76-year-old man presents to the Emergency Department 8 hours after the sudden onset of right arm weakness
and problems with word finding. Past medical history is positive for type II diabetes mellitus. Medications
include glyburide and metformin. On examination his blood pressure is 180/100 mm Hg, heart rate 60 and
regular. Neurological examination confirms an expressive aphasia and upper motor neuron weakness in the
right arm. Computerized tomography scan of the head is negative. Which one of the following is the most likely
diagnosis?

Right anterior cerebral artery (ACA) cerebrovascular accident


Lacunar cerebrovascular accident
Left anterior cerebral artery (ACA) cerebrovascular accident
Right middle cerebral artery (MCA) cerebrovascular accident
Left middle cerebral artery (MCA) cerebrovascular accident

Question: 20

A 72-year-old man with schizophrenia was switched to haloperidol 6 months ago. On a review visit, his wife
complains that he has had episodes of drooling and has had difficulty feeding himself. Neurologic examination
reveals marked stiffness and a new bilateral moderate tremor. Which one of the following is the most likely
cause of these symptoms?

Parkinsonism.
Dystonic reaction.
Tardive dyskinesia.
Neuroleptic malignant syndrome.
Acute akathisia.
Question: 21

A previously well 15-year-old boy is admitted to the Intensive Care Unit with a pneumococcal empyema that
required a chest tube drainage for 5 days and the administration of intravenous antibiotics. Current blood tests
reveal:

Sodium 140 mmol/L


Potassium 4 mmol/L
Bicarbonate 25 mmol/L
Creatinine 75 mol/L
Urea 5.3 mmol/L
Calcium 2.75 mmol/L
Phosphate 1.5 mmol/L
Albumin 32 g/L

Urinary calcium to creatinine ratio is 1.5 (normal < 0.7) mmol/mmol. Which one of the following is the most
likely cause of the hypercalcemia?

Malignancy
Hyperparathyroidism
Immobilization
Hypervitaminosis D
Sarcoid disease

Question: 22

Your clinic provides yearly periodic health examinations to all patients over the age of 65. Which one of the
following clinical situations most warrants screening for depression?

Individuals who have gained more than 10 kg in the last year.


Recent admission/placement in a nursing home/long term care home.
Individuals who have retired from work in the last year.
Hospitalized individuals who develop paranoid delusions and agitation after surgery.
Individuals who have had falls in the last year.

Question: 23

A 28-year-old stockbroker presents with a painless left scrotal mass which he noticed 8 weeks ago. He denies
any injury. Physical examination finds a non-tender testicular mass that does not transilluminate. Which one of
the following is the most indicated at this time?

Needle biopsy.
Repeat examination in 6 weeks.
Open biopsy.
Ultrasound.
Computed tomography scan.
Question: 24

A 50-year-old woman presents to your office because of pain in her left hip. X-rays of the hip show the
hemipelvis is thickened and mottled in appearance. The cortical bone outline is enlarged. The blood work,
including calcium and liver function tests, is normal except that the alkaline phosphatase is markedly elevated.
The most likely underlying cause is

Paget disease.
fibrous dysplasia.
chronic osteomyelitis.
osteomalacia.
metastatic cancer of the bone.

Section: 3

Question: 1

An adult man presents with the condition seen in the referenced illustration. Which one of the following is NOT
associated with this finding?

chronic bronchitis.
carcinoma of the lung.
infective endocarditis.
bronchiectasis.
right to left cardiac shunt.

Question: 2

A male cirrhotic patient died after a lengthy hospital stay. Autopsy revealed him to have a small, fibrotic liver, 8
cm wide at mid-point. Prior to his death, several clinicians, with varying years of clinical experience, had
estimated his liver size to be larger (average 15 cm, range 13 to 17 cm). Clinician performance illustrates

clinical disagreement.
good reliability, poor validity.
good inter-observer reliability, poor intra-observer reliability.
post-hoc measurement bias.
good validity, poor reliability.
Question: 3

A 48-year-old man walks home after his snowmobile breaks down. He is complaining of having a painful right
foot. The following image demonstrates the appearance of his foot. Which one of the following treatments is
correct?

Massage the injured part.


Rewarm in a water bath at 40C.
Plan early surgical debridement.
Immediate ambulation.
Leave blisters intact.

Question: 4

Which one of the following procedures is most likely to confirm the diagnosis of an endometrial polyp?

Endometrial biopsy.
Laparoscopy.
Dilatation and curettage.
Hysteroscopy.
Pelvic ultrasound

Question: 5

A 6-year-old boy is admitted with a depressed level of consciousness, fever and a purpuric rash that appeared 5
hours previously. Twelve hours after initial fluid resuscitation and treatment with intravenous antibiotics, his
plasma sodium is 120 mmol/L. A diagnosis of inappropriate antidiuretic hormone secretion (SIADH) would be
best supported by which one of the following?

Urine osmolality of 150 mmol/kg and fractional excretion of sodium of 2%.


Urine osmolality of 600 mmol/kg and fractional excretion of sodium of 2%.
Urine osmolality of 100 mmol/kg and fractional excretion of sodium of 0.75%.
Urine osmolality of 450 mmol/kg and fractional excretion of sodium of 0.25%.
Urine osmolality of 300 mmol/kg and fractional excretion of sodium of 0.5%.

Question: 6

A 57-year-old man has been admitted to the coronary care unit for a myocardial infarction. You are called to
evaluate him because he has become agitated, disoriented, and seems to be hallucinating. Which one of the
following medications is the most appropriate at this time?

Haloperidol.
Lithium carbonate.
Phenobarbital.
Chlorpromazine.
Morphine sulfate.
Question: 7

An 18-year-old man presents with yellow-white discoloration and thickening of his toenails, associated with
collection of keratotic debris under two of his nails. What is the most useful test to confirm the diagnosis?

Wood lamp examination.


Nail scraping and culture.
X-ray of the toes.
Direct immunofluorescence.
Nail biopsy.

Question: 8

A 65-year-old man presents with third degree hemorrhoids. Which one of the following is the best treatment?

Cryotherapy
Stool softener
Banding
Topical anti-inflammatories
Laser

Question: 9

Within a cohort study, each group of people must be

exposed to the same disease.


born in the same year.
diagnosed with the same disease.
followed over a period of time.
resident in the same geographic area.

Question: 10

A 54-year-old menopausal woman consults you regarding hormonal replacement. She smokes and has mild
hypertension. She previously declined hormonal replacement because her grandmother died of breast cancer.
She should be advised of which one of the following?

Her risk of endometrial cancer is increased on combined replacement.


Her risk of dying of heart disease is much greater than that of breast cancer.
Her family history is a contraindication to estrogen replacement.
Hypertension is a contraindication to estrogen replacement.
Smoking is not associated with an increased risk of osteoporosis.
Question: 11

You are discussing a "do not resuscitate (DNR)" order with a patient who has disseminated pancreatic cancer.
Even after explaining that cardiopulmonary resuscitation (CPR) will not be effective in her case, she insists that
everything possible should be done if she has a cardiac arrest, including CPR. Which one of the following is the
most appropriate response?

Withhold the DNR order, but discuss with the nurses that a "slow code" response to cardiac arrest
would be appropriate.
Withhold the DNR order and apply CPR if cardiac arrest occurs.
Write a DNR order.
Arrange for an ethics consultation with the care team and the patient.
Transfer the patient's care to a colleague.

Question: 12

A 17-year-old woman presents with primary amenorrhea. She reports normal breast development started at 11-
years-old and has a normal distribution of body hair. Physical exam confirms normal body habitus. Height is
162 cm. Weight is 55 kg. Pelvic exam reveals a normal female escutcheon and normal labia. On separating the
labia, you see a bluish tense bulge. A rectal exam confirms a midline mass. Which one of the following is the
appropriate management?

Hormonal profile
Bone age
Refer for surgery.
Computerized tomography of the sella turcica
Ultrasound pelvis

Question: 13

In which one of the following situations would an ethical physician report concerns about a colleague to the
provincial/territorial medical licensing authority?

She drinks alcohol most weekends when not on call.


She is in an intimate relationship with a current patient.
She is taking multiple prescription and non-prescription medications.
She has confided to you that she is under psychiatric care.
She has frequent fits of temper.

Question: 14

A 22-year-old woman undergoes a local excision of a vulvar lesion suspicious of melanoma. Prior to the
procedure, she received 5 mg of diazepam orally and a total of 8 mL of lidocaine without epinephrine, injected
subcutaneously. When she attempts to sit up after the procedure, she reports dizziness and loses consciousness.
Her blood pressure is 80/40 mm Hg and her pulse is 45/minute. Which one of the following is the most likely
cause of her symptoms?

Allergic reaction to diazepam


Enlarging vulvar hematoma with significant hypovolemia.
Hypovolemia due to preoperative fasting.
Vasovagal reaction to the procedure.
Toxic reaction to lidocaine.
Question: 15

A 10-year-old boy was placed on risperidone for the treatment of severe Tourette disorder. You inform the boy
and his father of potential side effects and indicate that you will have to monitor this boy's response to treatment
regularly. Which one of the following would you need to monitor at each visit?

Weight
Cholesterol level
Calcium
Renal function
Complete blood count

Question: 16

A 63-year-old woman presents to your office with left groin pain and difficulty with ambulation. An x-ray
shows moderate osteoarthritic changes in both hips (joint space narrowing and osteophytes). She has no
allergies and has not tried any non-pharmacotherapy or pharmacotherapy. Which one of the following is the
most appropriate next step?

Nonsteroidal anti-inflammatories
Acetaminophen
Glucosamine
Surgical consult
Computerized tomography scan

Question: 17

An active 70-year-old man presents with a two-month history of slowly progressive and painless edema of his
entire left leg. He is otherwise asymptomatic. Your physical examination shows normal abdominal examination,
a small hard prostate gland, diminished but symmetric arterial pulses in the legs, no calf tenderness and no
evidence of adenopathy. Which of the following is the most likely diagnosis?

Deep vein thrombosis.


Metastatic prostate cancer.
Congestive heart failure.
Arterial insufficiency.
Varicose veins.
Question: 18

A 62-year-old woman weighing 65 kg presents with a 4-month history of increasing abdominal girth, bilateral
leg swelling and recent development of orthopnea. She is not taking any medications, drinks no alcohol and has
never been hospitalized. She appears deeply tanned, her sclerae are mildly icteric, her liver span is 14 cm in the
mid-clavicular line, and the spleen is palpable. Laboratory results reveal:

Hemoglobin 98 g/L
White blood cell count 3.2 x 109/L
Platelet count 80 x 109/L
Serum ferritin 1200 mg/L (N = 18-200 mg/L)
Hemoglobin 98 g/L

Which one of the following tests would be most helpful in establishing the etiology of this condition?

Stool examination for occult blood.


Reticulocyte count.
Bone marrow aspiration.
Arterial blood gases.
Serum iron and total iron-binding capacity.

Question: 19

You are asked to assess a 15-year-old girl who is living in a group home because of behavior problems
(stealing, vandalism, skipping school and fighting). A diagnosis of conduct disorder is made. Which one of the
following co-morbid disorders would you most likely find upon further assessment?

Eating disorder.
Schizophrenia.
Obsessive compulsive disorder.
Personality disorder.
Attention deficit with hyperactivity disorder.

Question: 20

A 57-year-old woman is seen in the Emergency Department in acute distress with severe squeezing chest pain,
diaphoresis, marked anxiety and unable to swallow. Her husband states her symptoms came on acutely while
eating a meal in a Thai restaurant of chicken and peanuts. She is given sublingual nitroglycerin and her
symptoms abate within 15 minutes. Which one of the following is the most likely diagnosis?

Aspiration
Anaphylaxis
Globus hystericus
Angina
Esophageal spasm
Question: 21

A 32-year-old man presents to your office with a 3-day history of pain with swallowing. He has been feeling
unwell with a non-productive cough and no fever. On physical exam he has a reddened pharynx, slightly
swollen tonsils with no pustular exudates. There is no cervical lymphadenopathy. Which one of the following
organisms is the most likely causative agent?

Ebstein Barr virus


Streptococcus pneumoniae
Mycoplasma pneumoniae
Cytomegalovirus
Rhinovirus

Question: 22

A 25-year-old woman is arrested by police outside a local bar. She is confused and combative, smells of
alcohol, and is known to police as an individual with a history of alcohol and intravenous drug use, and prior
arrests for being intoxicated in a public place. She is placed in a police holding cell. She is found 3 hours later
in cardiac arrest. Attempts at cardiac resuscitation are started, and she is transferred to your hospital. After 1
hour, the attempts at resuscitation are terminated, and she is pronounced dead. Which one of the following
would be the most correct action?

Contact the family to arrange a post-mortem examination to confirm the cause of death.
List cardiac arrest as the cause of death on the death certificate.
Contact the medical examiner/coroner.
List alcohol and drug abuse as the cause of death on the death certificate.
List drug overdose (specific unknown) as the cause of cardiac arrest on the death certificate.

Question: 23

A previously well 1-month-old infant presents in the Emergency Department with fussiness. His vitals signs
are: temperature 37.1 rectally, respiratory rate 50/min, heart rate 235/ min and blood pressure 85/35. He is well-
perfused. Which one of the following investigations should be performed first?

Serum calcium
Blood gas analysis
Echocardiogram
Electrocardiogram
Chest radiograph

Question: 24

A 32-year-old with a substance abuse disorder presents with a 3-month history of impairment of concentration
and memory associated with a deterioration of higher intellectual functions. Which one of the following tests is
the most appropriate?

Serum thyroid-stimulating hormone.


Lumbar puncture.
Neuropsychological evaluation.
Electroencephalogram.
Human immunodeficiency virus antibody titer.
Section: 4

Question: 1

A mother and her 17-year-old daughter presents to your office to discuss birth control. They ask for information
on human papilloma virus vaccination. Which one of the following would you advise regarding the
administration of this vaccine?

It should not be given to girls who have had genital warts in the past.
It should be given only to girls between the age of 9 and 13 who have not yet become sexually
active.
It should be offered to all girls aged 9 to 26 years of age.
It should not be given to girls over the age of 20 years of age regardless of sexual activity.
It should not be given to girls who have had previous Papanicolaou smear abnormalities.

Question: 2

An obese 61-year-old male collapsed with sudden pain at a shopping center and is brought to hospital by
ambulance. He is diaphoretic. His pulse is 96/minute; blood pressure, 100/70 mm Hg; he complains of severe
pain in his abdomen and left flank. Which one of the following is the most likely diagnosis?

Acute diverticulitis.
Acute hemorrhagic pancreatitis.
Volvulus of sigmoid colon.
Ruptured aortic aneurysm.
Mesenteric vascular occlusion.

Question: 3

A 28-year-old housewife presents with a 12-week history of intermittent episodes of slightly raised, itchy, skin
lesions on her trunk and limbs. For weeks, she has noticed increasing amounts of swelling around her lips and
tongue when she has had flare-ups of her rash. Which one of the following is the most appropriate statement for
this presentation?

Reassurance and cold compresses are adequate treatment.


Chronic urticaria is a benign process.
This patient's condition may be life-threatening.
It will respond well to antihistamines.
Laboratory testing is required to identify the cause.
Question: 4

An 85-year-old wheelchair bound woman comes in for renewal of blood pressure medication. Three months
ago, she moved in with her only son and his family following the death of her husband who used to take care of
her. On examination, she looks unkempt, has lost 4.5 kg and has multiple bruises on her upper arms. Her blood
pressure is 170/95 mm Hg. When questioned specifically about the bruises and how things are in general, she
becomes fearful and withdrawn. She admits to really missing her husband. Which one of the following is the
most indicative of a possibility of elder abuse?

Her lack of personal care.


Her abnormal blood pressure.
Her bereavement following her husband's death.
Her fearful and withdrawn state.
Her recent move to her son's house.

Question: 5

You have successfully treated a patient suffering from an acute intoxication of stimulant drugs. In your
discharge planning, which one of the following management options should you include to prevent relapse?

A prescription for disulfiram.


A prescription for diazepam 10 mg by mouth four times a day.
A course of naloxone.
Methadone maintenance treatment program.
Close daily monitoring by a support group.

Question: 6

As you are flying non-stop from Toronto to Vancouver, a flight attendant asks if there is a doctor on board to
see an ill passenger. Which one of the following considerations applies?

You have a moral duty to provide assistance.


You have a legal duty to respond under federal aviation law.
You could be held liable if you advise an emergency landing which turns out to be unnecessary.
You have a legal duty to respond if the illness is life-threatening.
You have a legal duty to provide assistance under the Good Samaritan Law.

Question: 7

Which of the following is an appropriate indication for induction of labor?

Term pre-labor rupture of membranes.


Estimated fetal weight of 4300 g.
Previous Cesarean delivery at 40 weeks gestation.
Maternal gestational diabetes at 37 weeks gestation.
Post-dates 40 4/7 weeks gestation.
Question: 8

When confronted with an adolescent presenting with an obesity problem, which one of the following would
mandate a more complete investigation?

Obese parents.
Sexual development at age 10.
Family history of diabetes.
Short stature.
Advanced bone age.

Question: 9

A mother brings her 9-year-old girl to your office with concerns about her academic performance. The girl is
good at speaking, reading, and writing but has difficulty counting and solving math problems. She also has
difficulty keeping score when playing cards. Which one of the following is the most appropriate intervention?

Recommend a visual assessment


Recommend tutoring
Suggest vitamin B complex supplementation
Prescribe atomoxetine
Recommend a psychoeducational assessment

Question: 10

A 60-year-old man with a history of ischemic heart disease and two previous myocardial infarctions is brought
to the Emergency Department unresponsive. His blood pressure is not detectable. The electrocardiogram
demonstrates ventricular fibrillation. He is intubated, and cardiopulmonary resuscitation is initiated. He is
placed on a monitor and an intravenous access is obtained. He is defibrillated at 200, 300 and 360 joules and
remains in ventricular fibrillation. What is the most appropriate next step?

Perform a pericardiocentesis to rule out cardiac tamponade.


Defibrillate again at 360 joules.
Administer 1 mg of intravenous atropine.
Administer 1 mg of intravenous epinephrine.
Administer 1.5 mg/kg of intravenous lidocaine.

Question: 11

A 24-year-old woman presents with a 10-day history of vaginal itching and a foul-smelling vaginal discharge.
On speculum examination, the yellow frothy discharge and other findings pictured are visualized. For the
patient, the next best step in management in order to confirm the diagnosis would be to

do vaginal cultures and plate on chocolate agar.


examine a hanging drop preparation under a microscope.
perform colposcopy.
do vulvar skin scrapings for fungus.
do paired herpes antibody titres.
Question: 12

Which one of the following is a physician legally required to do when transferring a trauma patient to another
location?

Talk to the ambulance attendants transferring the patient to ensure they understand the patient's
problems.
Ensure that there is a physician ready to receive the patient in the other location.
Notify the patient's next of kin.
Send the patient's clinical record or copy.
Accompany the patient.

Question: 13

A 24-year-old gravida II, para I presents at 14 weeks' gestation with a history of recurrent discharge and vaginal
bleeding. On examination, the uterine fundus is at the level of the umbilicus. The serum human chorionic
gonadotropin is 38,000 units and there is no identifiable fetus on ultrasound. Which one of the following is the
best management?

Prostaglandin amnio-infusion.
Hysterectomy.
Dilation and suction curettage.
Methotrexate.
Hysterotomy.

Question: 14

A 3-year-old child presents to your office with a history of progressively severe abdominal pain being treated
with a lead-containing folk remedy. Which one of the following is the most indicative of lead exposure?

Mild hypertension (150/100 mm Hg).


Mild microcytic hypochromic anemia.
Increased 24-hour urinary lead.
Blood lead level of 2 mmol/L (normal < 0.5 mmol/L).
Reduced total white blood cell count.

Question: 15

A 25-year-old woman presents with a 4-month history of spontaneous galactorrhea. She has developed irregular
menses and has never taken oral contraceptives. On physical examination she has no visual defect. A few drops
of milky fluid are expressed from both nipples. Plain films of the sella turcica are normal. Which of the
following would be most appropriate diagnostic test?

Pregnancy test.
Bilateral mammography.
Plasma cortisol.
Plasma prolactin.
Tomograms of the sella turcica.
Question: 16

A previously healthy 2-year-old boy presents to your walk in clinic with a 3-day history of non-bloody, non-
bilious vomiting followed by a 5-day history of non-bloody watery diarrhea. Today the diarrhea is bloody, and
he is not wetting his diapers. He attended a family picnic 10 days prior. His physical examination reveals a quiet
child with a heart rate of 100/minute and a blood pressure of 120/80 mm Hg. His mouth is moist, his eyes are
not sunken, and his skin turgor is normal. The rest of his examination is completely normal. Which one of the
following is the most important next step in investigation for this patient?

Coagulation screen
Supine and upright abdominal radiographs
Abdominal ultrasound
Air contrast enema
Complete blood count and creatinine

Question: 17

In Canada, the age-standardized male incidence rate decreased from 1993 through 2000 for which one of the
following cancers?

Lung.
Colorectal.
Thyroid.
Melanoma.
Non-Hodgkin's lymphoma.

Question: 18

You are asked to see a 4-year-old boy who has had intermittent pain in his legs for the past 6 months. The pain
is mainly in the thighs and calves that wakes the child at night. He is otherwise well. Which one of the
following is the most probable diagnosis?

Lyme disease.
Viral arthritis.
Aseptic necrosis.
Growing pains.
Bone malignancy.

Question: 19

The drug of choice in the treatment of a tremor secondary to lithium therapy is

propranolol.
lorazepam.
amantadine.
chlorpromazine.
benztropine.
Question: 20

A 32-year-old woman presents to your office requesting nutritional advice for her husband and herself. Both are
in good health and have made a commitment to a healthier lifestyle through diet, exercise and cessation of
smoking. Nutritional recommendations suggest that 55% of total energy requirements should be obtained from
which one of the following food sources?

Saturated fat.
Protein.
Unsaturated fat.
Carbohydrate.
Fiber.

Question: 21

This 2-week-old infant has a localized skin eruption on the face. What would be the appropriate management of
this condition?

Arrange consultation with a dermatologist.


Culture exudate from lesion and observe.
Explain the benign cause of this condition to the parents.
Culture exudate and initiate acyclovir.
Culture exudate and initiate intravenous antibiotics.

Question: 22

A 73-year-old woman presents with a 10-day history of lower abdominal pain and constipation alternating with
diarrhea. There is urinary frequency. On examination, she has diffuse lower abdominal tenderness with signs
much worse suprapubically and in the left iliac fossa. Her temperature is 38.8C and her white blood cell count
is 18 x 109/L. Which one of the following diagnoses is the most likely?

Acute diverticulitis.
Carcinoma of the sigmoid colon.
Pelvic inflammatory disease.
Non-hemorrhagic cystitis.
Ischemic colitis.

Question: 23

Which one of the following is recommended for patients with adult celiac disease?

Boiled rice.
Rye bread.
Whole wheat bread.
Oat-containing dry cereal.
Barley and beef soup.
Question: 24

The referenced illustration shows a 2-year-old child who has an unhappy disposition with anorexia and weight
loss present for about 6 months. His hemoglobin is 106 g/L. Which investigation would be most helpful in
making a diagnosis?

Liver biopsy
Ultrasound of abdomen
Chest x-ray
Bone marrow biopsy
Small bowel biopsy
Clinical Laboratory Tests
Normal Values

This table lists reference values for the most common


laboratory tests and is intended for interpretation of the results
as they are provided in the examinations. Note that all values
are provided in SI units. All values apply to adults.

Many important laboratory reference values are not listed


here, because of the less frequent use of these tests. Such
values are inserted in parentheses following the result
recorded in the examination questions.

Blood Cerebrospinal Fluid


Coagulation Urine
Hemogram
Chemical Constituents

Blood Back to Top


COAGULATION (HEMOSTASIS)
Bleeding time (Ivy) <9 minutes
International Normalized Ratio (INR) 0.9-1.2
Partial thromboplastin time (PTT) 28-38 seconds
Prothrombin time (PT) 10-13 seconds
HEMOGRAM Back to Top
Hematocrit (Hct)
Female 0.370-0.460
Male 0.420-0.520
Hemoglobin (Hb)
Female 123-157 g/L
Male 140-174 g/L
Mean corpuscular volume (MCV) 80-100 fL
Mean corpuscular hemoglobin (MCH) 27-34 pg
Platelet count 130-400 X 109/L
Red blood cells (RBC)
Female 4.0-5.2 X 1012/L
Male 4.4-5.7 X 1012/L
Red cell distribution width (RDW) 11.5-14.5%
Reticulocyte count 20-84 X 109/L
Sedimentation rate (Westergren) <15 mm/hour
White blood cells & differential
White blood cell count (WBC) 4-10 X 109/L
Segmented neutrophils 2-7 X 109/L
Band neutrophils <0.7 X 109/L
Basophils <0.10 X 109/L
Eosinophils <0.45 X 109/L
Lymphocytes 1.5-3.4 X 109/L
Monocytes <0.95 X 109/L
CHEMICAL CONSTITUENTS Back to Top
Albumin (serum) 35-50 g/L
Alkaline phosphatase (serum) 35-100 U/L
Aminotransferase (transaminase) (serum)
Alanine (ALT; SGPT) 3-36 U/L
Aspartate (AST; SGOT) 0-35 U/L
Gamma glutamyl transferase
Female 10-30 U/L
Male 10-35 U/L
Amylase (serum) <160 U/L
Bicarbonate (HCO3) (serum) 24-30 mmol/L
Bilirubin (serum)
Direct (conjugated) <7 mol/L
Total <26 mol/L
Calcium (serum)
Total 2.18-2.58 mmol/L
Ionized 1.05-1.30 mmol/L
Chloride (serum) 98-106 mmol/L
Cholesterol (serum) <5.2 mmol/L
High density lipoprotein >0.9 mmol/L
Cortisol 160-810 mmol/L
Creatine kinase (CK) (serum) 5-130 U/L
Creatinine (serum)
Female 50-90 mol/L
Male 70-120 mol/L
Glucose fasting (serum) 3.3-5.8 mmol/L
Hemoglobin A1C 4-6%
Lactate dehydrogenase (LDH) (serum) 95-195 U/L
Lipase (serum) <160 U/L
Magnesium (serum) 0.75-0.95 mmol/L
Osmolality (serum) 280-300 mmol/kg
Oxygen saturation (arterial blood) (SaO2) 96-100%
PaCO2 (arterial blood) 35-45 mm Hg
PaO2 (arterial blood) 85-105 mm Hg
pH 7.35-7.45
Phosphorus (inorganic) (serum) 0.8-1.5 mmol/L
Potassium (serum) 3.5-5.0 mmol/L
Protein (serum)
Total 60-80 g/L
Albumin 35-50 g/L
Sodium (serum) 135-145 mmol/L
Thyroid-stimulating hormone (sensitive) 0.4-5.0 mU/L
Thyroine (free T4) 11-23 pmol/L
Transaminase - see Aminotransferase
Triglycerides (serum) <2.20 mmol/L
Troponin T (TnT) <0.10 g/L
Urea (BUN) (serum) 2.5-8.0 mmol/L
Uric acid (serum) 180-420 mol/L
Cerebrospinal Fluid Back to Top
Cell count <4 x 106/L
Glucose 2-4 mmol/L
Proteins (total) 0.20-0.45 g/L
Urine Back to Top
Calcium <7.3 mmol/day
Chloride 110-250 mmol/day
Creatinine 6.2-17.7 mmol/day
Osmolality 100-1200 mmol/kg
Potassium 25-120 mmol/day
Protein <0.15 g/day
Sodium 25-260 mmol/day

S-ar putea să vă placă și